PT 2 MCQ's (probs for unit 3/4??)

Ace your homework & exams now with Quizwiz!

(A) DNA ligase (B) DNA polymerase (C) RNA polymerase (D) Restriction enzyme (E) Reverse transcriptase Enzyme used during replication to attach Okazaki fragments to each other A DNA ligase B DNA polymerase C RNA polymerase D Restriction enzyme E Reverse transcriptase

A

(A) Glysolysis (B) Krebs cycle (citric acid cycle) (C) Calvin cycle (light-independent reactions of photosynthesis) (D) Light-dependent reactions of photosynthesis (E) Chemiosmosis Process in which sugar is oxidized to pyruvic acid A B C D E

A

(A) Transcription(B) Translation(C) Transformation(D) Replication(E) Reverse transcription Process in which RNA is produced by using a DNA template A B C D E

A

1/1 MC point When a substance moves across the plasma membrane along a concentration gradient at a rate faster than would be expected by simple diffusion alone but without the expenditure of metabolic energy, the process is best described as A facilitated diffusion B osmosis C active transport D pinocytosis E the work of an ion pump

A

1/1 MC point Which of the following correctly illustrates a dipeptide and an amino acid in the optimal position to form a tripeptide? Chem of life # 23 A B C D 23

A

A respirometer is a container used to measure the amount of oxygen consumed by an organism. A respirometer was used to determine how environmental temperature affects the uptake of oxygen in one 300-gram rat and one 50-gram mouse. The results of this experiment are shown on the graph below. CR quiz #34 Which of the following hypotheses is best supported by the results of this experiment? A Metabolic rate per gram of tissue is higher in smaller mammals. B Metabolic rate per gram of tissue is not related to body mass. C Mice produce less CO2 per gram of tissue than do rats. D Rats consume more food per gram of tissue than do mice. E Rats lose more heat per gram of tissue than do mice.

A

A student claims that the Y chromosome contains the sex-determining region gene, known as the SRY gene, which causes male fetuses to develop testes. Which of the following provides correct information about cell signaling that supports the claim? A The SRYSRY gene produces a protein that binds to specific regions of DNADNA in certain tissues, which affects the development of these tissues. B The SRYSRY gene produces a protein that deletes portions of the XX chromosome in males so that male characteristics can develop. C The SRYSRY gene produces an RNARNA segment that is exported from specific cells and targets the developing gonads. D The SRYSRY gene is found only in tissues of the developing gonads.

A

A student placed 20 tobacco seeds of the same species on moist paper towels in each of two petri dishes. Dish A was wrapped completely in an opaque cover to exclude all light. Dish B was not wrapped. The dishes were placed equidistant from a light source set to a cycle of 14 hours of light and 10 hours of dark. All other conditions were the same for both dishes. The dishes were examined after 7 days, and the opaque cover was permanently removed from dish A. Both dishes were returned to the light and examined again at 14 days. The following data were obtained. Photosynthesis quiz #2 According to the results of this experiment, germination of tobacco seeds during the first week is A increased by exposure to light B unaffected by light intensity C prevented by paper towels D accelerated in green-leaved seedlings

A

An experiment to measure the rate of respiration in crickets and mice at 10o C and 25o C was performed using a respirometer, an apparatus that measures changes in gas volume. Respiration was measured in mL of O2 consumed per gram of organism over several five-minute trials, and the following data were obtained. CR topic questions # 5 According to the data, the mice at 10o C demonstrated greater oxygen consumption per gram of tissue than did the mice at 25o C. This is most likely explained by which of the following statements? A The mice at 10o C had a higher rate of ATP production than the mice at 25o C. B The mice at 10o C had a lower metabolic rate than the mice at 25o C. C The mice at 25o C weighed less than the mice at 10o C. D The mice at 25o C were more active than the mice at 10o C.

A

An experiment to measure the rate of respiration in crickets and mice at 10o C and 25o C was performed using a respirometer, an apparatus that measures changes in gas volume. Respiration was measured in mL of O2 consumed per gram of organism over several five-minute trials, and the following data were obtained. During aerobic cellular respiration, oxygen gas is consumed at the same rate as carbon dioxide gas is produced. In order to provide accurate volumetric measurements of oxygen gas consumption, the experimental setup should include which of the following? CR quiz #5 A A substance that removes carbon dioxide gas B A plant to produce oxygen C A glucose reserve D A valve to release excess water

A

An experiment to measure the rate of respiration in crickets and mice at 10oC and 25oC was performed using a respirometer, an apparatus that measures changes in gas volume. Respiration was measured in mL of O2 consumed per gram of organism over several five-minute trials and the following data were obtained. CR topic questions # 4 According to the data, the mice at 10oC demonstrated greater oxygen consumption per gram of tissue than did the mice at 25oC. This is most likely explained by which of the following statements? A The mice at 10oC had a higher rate of ATP production than the mice at 25oC. B The mice at 10oC had a lower metabolic rate than the mice at 25oC. C The mice at 25oC weighed less than the mice at 10oC. D The mice at 25oC were more active than the mice at 10oC.

A

Anabaena is a simple multicellular photosynthetic cyanobacterium. In the absence of fixed nitrogen, certain newly developing cells along a filament express genes that code for nitrogen-fixing enzymes and become nonphotosynthetic heterocysts. The specialization is advantageous because some nitrogen-fixing enzymes function best in the absence of oxygen. Heterocysts do not carry out photosynthesis but instead provide adjacent cells with fixed nitrogen, in exchange receiving fixed carbon and reduced energy carriers. Cell signaling quiz #3 As shown in the diagram above, when there is low fixed nitrogen in the environment, an increase in the concentration of free calcium ions and 2-oxyglutarate stimulates the expression of genes that produce two transcription factors (NtcA and HetR) that promote the expression of genes responsible for heterocyst development. HetR also causes production of a signal, PatS, that prevents adjacent cells from developing as heterocysts. Based on your understanding of the ways in which signal transmission mediates cell function, which of the following predictions is most consistent with the information given above? A In an environment with low fixed nitrogen, treating the Anabaena cells with a calcium-binding compound should prevent heterocyst differentiation. B A strain that overexpresses the patS gene should develop many more heterocysts in a low fixed nitrogen environment. C In an environment with abundant fixed nitrogen, free calcium levels should be high in all cells so that no heterocysts develop. D In environments with abundant fixed nitrogen, loss of the hetR gene should induce heterocyst development.

A

Analysis of DNA sequences from two individuals of the same species results in a greater estimate of genetic variability than does analysis of amino acid sequences from the same individuals because A different DNA sequences can code for the same amino acid B some amino acid variations cannot be detected by protein electrophoresis C DNA sequencing is a more reliable technique than protein electrophoresis D proteins are more easily damaged than is DNA E DNA is more heat-sensitive and therefore varies more

A

Antidiuretic hormone (ADH) is important in maintaining homeostasis in mammals. ADH is released from the hypothalamus in response to high tissue osmolarity. In response to ADH, the collecting duct and distal tubule in the kidney become more permeable to water, which increases water reabsorption into the capillaries. The amount of hormone released is controlled by a negative feedback loop. Based on the model presented, which of the following statements expresses the proper relationship between osmolarity, ADH release, and urine production? A As tissue osmolarity rises, more ADH is released, causing less water to be excreted as urine. B As tissue osmolarity rises, less ADH is released, causing less water to be excreted as urine. C As tissue osmolarity rises, more ADH is released, causing more water to be excreted as urine. D As tissue osmolarity rises, less ADH is released, causing more water to be excreted as urine.

A

Assume that genes A and B are not linked. If the probability of allele A in a gamete is 1/2 and the probability of allele B in a gamete is 1/2, then the probability that both A and B are in the same gamete is A 1/2 x 1/2 B 1/2 + 1/2 C (1/2) / (1/2) D 1/2−−−√ E |1/2 — 1/2|

A

Cell communication unit test # 10 Insulin, a hormone secreted by pancreatic cells, stimulates glucose uptake in skeletal muscle cells by mobilizing glucose transporter proteins (GLUT4) to the plasma membrane. As depicted in Figure 1, binding of insulin to the insulin receptor triggers an intracellular signaling cascade in which certain molecules activate other molecules in a relay of the hormone signal to cell targets. One outcome of the signaling cascade is mobilization of GLUT4 from vesicle storage sites in the cytoplasm to sites at the cell surface, where GLUT4 allows glucose to enter the cell. In type 2 diabetes, the cellular response to insulin is disrupted, and individuals with type 2 diabetes cannot properly regulate their blood glucose levels. In an investigation of the insulin signaling pathway, samples of skeletal muscle were isolated from individuals who have type 2 diabetes and from individuals who do not. The results of several experiments that were performed on the muscle samples are shown in Figure 2, Figure 3, and Figure 4. Which of the following is a valid interpretation of the experimental results that explains how individuals with type 2 diabetes differ from individuals without diabetes? A The relatively low levels of glucose uptake in individuals with type 2 diabetes indicate that mobilization of GLUT4 to the cell surface is reduced in muscle cells of those individuals. B The relatively low levels of glucose uptake in individuals with type 2 diabetes indicate that no functional GLUT4 protein is produced in the muscle cells of those individuals. C The absence of activated insulin receptors in individuals with type 2 diabetes indicates that no insulin is secreted by the pancreatic cells of those individuals. D The absence of activated IRS-1 in individuals with type 2 diabetes indicates that no functional insulin receptor protein is produced in the muscle cells of those individuals.

A

Cell communication unit test #9 Insulin, a hormone secreted by pancreatic cells, stimulates glucose uptake in skeletal muscle cells by mobilizing glucose transporter proteins (GLUT4) to the plasma membrane. As depicted in Figure 1, binding of insulin to the insulin receptor triggers an intracellular signaling cascade in which certain molecules activate other molecules in a relay of the hormone signal to cell targets. One outcome of the signaling cascade is mobilization of GLUT4 from vesicle storage sites in the cytoplasm to sites at the cell surface, where GLUT4 allows glucose to enter the cell. In type 2 diabetes, the cellular response to insulin is disrupted, and individuals with type 2 diabetes cannot properly regulate their blood glucose levels. In an investigation of the insulin signaling pathway, samples of skeletal muscle were isolated from individuals who have type 2 diabetes and from individuals who do not. The results of several experiments that were performed on the muscle samples are shown in Figure 2, Figure 3, and Figure 4. Based on the information presented, which of the following genetic changes in an individual without diabetes is most likely to result in a disrupted cellular response to insulin signaling similar to that of an individual with type 2 diabetes? A A deletion in the gene encoding the insulin receptor that removes only the cytoplasmic domain of the protein B Duplication of the gene encoding a PI-3 kinase that results in synthesis of a muscle-specific variant of the enzyme in skin cells as well as in muscle cells C A mutation in the gene encoding IRS-1 that causes the protein to be active in muscle cells even in the absence of insulin signaling D Insertion of a small segment of DNA into the promoter of the Glut4 gene that results in increased synthesis of GLUT4 proteins in muscle cells

A

Cell cycle questions #9 A group of researchers cultured yeast cells in a nutrient-rich environment and a nutrient-poor environment and observed the duration of the stages of their cell cycles. The results of their study are summarized in Table 1. Table 1. Duration (in minutes) of yeast cell cycle phases in a nutrient-rich environment and a nutrient-poor environment Cell Cycle StagesNutrient-rich environmentNutrient-poor environmentS and G22333Mitosis4861G12057Cytokinesis510 The cell cycle of yeast cells grown in the nutrient-poor environment is approximately what percent of the cell cycle of yeast cells grown in the nutrient-rich environment? A 168 B 127 C 179 D 160

A

Cell signaling quiz # 13 Which of the following diagrams best represents hormone-activated gene expression? A B C D

A

Cell signaling quiz # 18 The epinephrine signaling pathway plays a role in regulating glucose homeostasis in muscle cells. The signaling pathway is activated by the binding of epinephrine to the beta-2 adrenergic receptor. A simplified model of the epinephrine signaling pathway is represented in Figure 1. Figure 1. A simplified model of the epinephrine signaling pathway in muscle cells Which of the following outcomes will most likely result from the irreversible binding of GDP to the G protein? A The intracellular concentration of glycogen will increase. B The intracellular concentration of activated protein kinase A will increase. C The intracellular concentration of cyclic AMPAMP will increase. D The intracellular concentration of glucose-1-phosphate will increase.

A

Epinephrine is a protein hormone found in many animals. Epinephrine stimulates a signaling pathway that results in the breakdown of glycogen to glucose in the liver cells. Which of the following describes the initial steps in the process whereby epinephrine stimulates glycogen breakdown? A Epinephrine binds to a cell-surface receptor; the activated receptor stimulates production of the second messenger, cAMP B Epinephrine binds to a cell-surface receptor; the activated receptor catalyzes the conversion of glycogen to glucose. C Epinephrine diffuses through the plasma membrane; the hormone dimerizes in the cytosol. D Epinephrine is taken into the cell by endocytosis; glycogen is converted to glucose in the endocytotic vesicle.

A

Eye color in a particular strain of fly is influenced by one gene with two alleles: a dominant allele that results in red eyes and a recessive allele that results in sepia eyes. A red-eyed female from a true-breeding population is mated with a sepia-eyed male. The F1 offspring are all red-eyed. The F1 flies are allowed to interbreed, producing the following in the F2 generation. Females: 40 red eyes; 13 sepia eyes Males: 39 red eyes; 11 sepia eyes Which of the following best describes the likely mode of inheritance for the eye-color gene? A The eye-color gene is likely autosomal because males and females have similar phenotype ratios. B The eye-color gene is likely autosomal because more females have sepia eyes than males do. C The eye-color gene is likely sex-linked because the males and females have similar phenotype ratios. D The eye-color gene is likely sex-linked because the males and females display both phenotypes.

A

For following group of questions first study the description of the data and then choose the one best answer to each question following it and fill in the corresponding oval on the answer sheet. To study the actions of the enzyme catalase on hydrogen peroxide, students performed the following experiment. Catalase was extracted from potatoes by blending raw potatoes in a blender with cold distilled water. The filtrate was stored on ice. The following hydrogen peroxide solutions were made: 1 percent, 5 percent, 10 percent, and 15 percent. Filter paper disks were soaked in the catalase filtrate and dropped into beakers containing the various solutions. The activity of the enzyme was measured by the amount of time it took for the disks to float to the surface of the solution on the bubbles produced by the reaction. The following data were obtained. Energy Unit Test #11 Which of the following experimental designs should the students use as a control for the experiment? A Place a catalase-soaked disk in a beaker of water. B Drop the disks from different heights into the solution. C Poke the floating disks back down to the bottom of the beaker and retime the rise of the disks. D Shake the beakers during the time the disks are rising. E Dip the disks in the 5% solution before putting them in any other solution.

A

For following group of questions first study the description of the situation and diagrams and then choose the one best answer to each question following it and fill in the corresponding oval on the answer sheet. A student studied the effects of light intensity on oxygen production in green algae. The algae were suspended in water inside a sealed glass jar, and the jar was placed into a constant-temperature, lightproof box containing a light source. A probe was inserted into the jar to record the concentration of oxygen. The probe was connected to a recording device. The setup is shown below. The student decreased the intensity of the light hourly and recorded the corresponding changes in oxygen concentration. The graph below shows the results from the recording device. An increase in the rate of oxygen production by algae would be accompanied by a comparable increase in the rate of production of which of the following substances? Photosynthesis quiz #4 A C6H12O6 B CO2 C NH3 D H2O

A

For the following questions: Graphs I-IV depict the effect of pH on the activity of four different hydrolytic enzymes. Energy unit test #8 The most likely explanation for the results shown in Graph I is that A pH affects the shape of the active site of the enzyme B pH affects the temperature of the reaction C the enzyme has a quaternary structure D the enzyme has disulfide bonds E pH affects the primary structure of the enzyme

A

Glycogen synthetase kinase 3 beta is a protein kinase that has been implicated in many types of cancer. Depending on the cell type, the gene for glycogen synthetase kinase 3 beta (GSK3β) can act either as an oncogene or as a tumor suppressor. Which of the following best predicts how GSK3β mutations can lead to the development of cancer? A Cells with inactive GSK3βGSK3β fail to trigger apoptosis. B Cells with inactive GSK3βGSK3β fail to proceed past the G2/MG2/M checkpoint. C Cells with overactive GSK3βGSK3β are more likely to repair DNADNA damage. D Cells with overactive GSK3βGSK3β have longer cell cycles.

A

Hereditary unit test #10 Table I shows the results of breeding experiments to examine the inheritance of flower color (purple versus white) and pod shape (inflated versus constricted). For the crosses recorded in Table I, true-breeding parents were crossed to produce F1 offspring, which were then testcrossed to homozygous recessive individuals. Table II shows the results of computer-simulated crosses to model the inheritance of leaf shape (broad versus narrow) and flower color (purple versus white). Based on the data in Table I, which of the following best explains why there are no individuals with constricted pods in the F1generation? A Inflated pod shape is dominant to constricted pod shape. B The inflated-pod offspring in the F1 generation are homozygous. C Constricted pod shape typically arises from a new mutation in the F1 generation. D The constricted-pod offspring are carriers for the inflated pod shape allele. Related Content & Skills Topic5.3 SkillSkill 4.B

A

Hereditary unit test #27 Directions: Each group of questions below concerns an experimental or a laboratory situation. In each case, first study the description of the situation. Then choose the one best answer to each question following it. In dogs, one pair of alleles determines coat color (dark and albino). Another pair of alleles determines hair length (short and long). Thus, each gamete will contain one of the coat-color alleles, C or c and one of the hair-length alleles, B or b. In repeated crosses of a specific dark, short-haired dog with an albino, long-haired dog, all the offspring were dark with short hair, as shown in cross I. However, in subsequent crosses of another dark, short-haired dog with a dark, long-haired dog, the ratios shown in cross II below were obtained. Which of the following correctly describes the relationship of the dark-coat-color allele to the albino condition? A It is dominant. B It is recessive. C It is codominant. D It is a polygenic inheritance pattern. E The alleles are linked.

A

In sweet pea plants, purple flower color is dominant over red flower color and long pollen grain shape is dominant over round pollen grain shape. Two sweet pea plants that are heterozygous for both flower color and pollen grain shape are crossed with one another. A geneticist is surprised to observe that there are far fewer round, purple offspring and long, red offspring than were predicted by the 9:3:3:1 expected phenotypic ratio. Which of the following statements provides the most reasonable prediction to account for the deviation from the expected results? A In sweet pea plants, the gene for flower color and the gene for pollen grain shape are genetically linked. B In sweet pea plants, the genes for flower color and for pollen grain shape both exhibit codominance. C Several mutations occurred spontaneously producing a deviation from the expected phenotypic ratios of the offspring. D The genes for flower color and pollen grain shape are inherited independently because of the law of independent assortment.

A

Intact cells of two unknown cell types were placed into solutions with different concentrations of NaCl. Type I cells swelled and burst in the solution with the lowest concentration of NaCl. Type II cells swelled but did not burst in the solution with the lowest concentration of NaCl. Which of the following descriptions of cell type I and cell type II are most consistent with the data? A Type 1: animal surrounded by plasma membrane Type 2: plant cell surrounded by a plasma membrane and cell wall B Type 1: plant surrounded by plasma membrane Type 2: bacteria cell surrounded by a cell wall C Type 1: plant surrounded by plasma membrane Type 2: animal surrounded by a plasma membrane and cell wall D Type 1: animal cell surrounded by a cell wall only Type 2: bacteria cell surrounded by a plasma membrane only

A

Meiosis topic questions #1 During prophase I replicated homologous chromosomes pair up and undergo synapsis. What testable question is generated regarding synapsis and genetic variability by Figure 1 ? A Is the distance between two gene loci related to crossover rate? B Does crossing over occur more often in some chromosomes than in others? C Is crossing over inhibited by methylation? D Is crossing over promoted by methylation?

A

Oxygen consumption can be used as a measure of metabolic rate because oxygen is A necessary for ATP synthesis by oxidative phosphorylation B necessary to replenish glycogen levels C necessary for fermentation to take place D required by all living organisms E required to break down the ethanol that is produced in muscles

A

Researchers propose a model to explain variation in phytoplankton cell sizes in a marine environment. They base their model on the idea that smaller cells absorb nutrients more efficiently. The researchers predict that the mean diameter of phytoplankton cells will change by 50 micrometers for every 5-kilometer increase in distance from the shore because of a gradual decrease in nutrient availability. To test their model, the researchers determine that the phytoplankton cells found closest to shore have a mean diameter of 900 micrometers. Based on the model, what will be the mean diameter of the phytoplankton cells that are found 25 kilometers from shore? 650 micrometers A 875 micrometers B 925 micrometers C 1150 micrometers

A

Scientists have found that DNA methylation suppresses crossing-over in the fungus Ascobolus immersus. Which of the following questions is most appropriately raised by this specific observation? A Is the level of genetic variation in the gametes related to the amount of DNADNA methylation observed? B Without crossing-over, will gametes be viable and be able to produce zygotes? C Does DNADNA methylation result in shorter chromosomes? D Is this species of fungus a diploid organism?

A

Simple cuboidal epithelial cells line the ducts of certain human exocrine glands. Various materials are transported into or out of the cells by diffusion. (The formula for the surface area of a cube is 6 X S2, and the formula for the volume of a cube is S3, where S = the length of a side of the cube.) Which of the following cube-shaped cells would be most efficient in removing waste by diffusion? A 10 cubed B 20 cubed C 30 cubed D 40 cubed

A

The energy required to run the Calvin cycle reactions of photosynthesis comes from which two substances produced during the light-dependent reactions? A ATP and NADPH B ADP and PO4 C H+ and PO2 D O2 and CO2 E H2O and CO2

A

The epinephrine signaling pathway plays a role in regulating glucose homeostasis in muscle cells. The signaling pathway is activated by the binding of epinephrine to the beta-2 adrenergic receptor. A simplified model of the epinephrine signaling pathway is represented in Figure 1. Cell signaling quiz review #6 Figure 1. A simplified model of the epinephrine signaling pathway in muscle cells Which of the following outcomes will most likely result from the irreversible binding of GDP to the G protein? A The intracellular concentration of glycogen will increase. B The intracellular concentration of activated protein kinase A will increase. C The intracellular concentration of cyclic AMPAMP will increase. D The intracellular concentration of glucose-1-phosphate will increase.

A

The figures below illustrate the similarities between ATP synthesis in mitochondria and chloroplasts. CR quiz # 27 The figures can best assist in answering which of the following questions? A Do electron transport chains create a gradient so that ATP synthase can generate ATP molecules? B What are the sources of energy that drive mitochondrial and chloroplast electron transport systems? C What is the optimal temperature at which ATP synthase chemically converts ADP and a phosphate group into one molecule of ATP? D What is the evolutionary relationship between the ATP synthase in mitochondria and the ATP synthase in chloroplasts?

A

The following questions refer to an experiment in which a dialysis-tubing bag is filled with a mixture of 3% starch and 3% glucose and placed in a beaker of distilled water, as shown below. After 3 hours, glucose can be detected in the water outside the dialysis-tubing bag, but starch cannot. If, instead of the bag, a potato slice were placed in the beaker of distilled water, which of the following would be true of the potato slice? A It would gain mass. B It would neither gain nor lose mass. C It would absorb solutes from the surrounding liquid. D It would lose water until water potential inside the cells is equal to zero. E The cells of the potato would increase their metabolic activity.

A

The following questions refer to an experiment in which a dialysis-tubing bag is filled with a mixture of 3% starch and 3% glucose and placed in a beaker of distilled water, as shown below. After 3 hours, glucose can be detected in the water outside the dialysis-tubing bag, but starch cannot. Which of the following best describes the condition expected after 24 hours? A The bag will contain more water than it did in the original condition. B The contents of the bag will have the same osmotic concentration as the surrounding solution. C Water potential in the bag will be greater than water potential in the surrounding solution. D Starch molecules will continue to pass through the bag. E A glucose test on the solution in the bag will be negative.

A

The following questions refer to an experiment that is set up to determine the relative volume of O2 consumed by germinating and nongerminating (dry) pea seeds at two different temperatures. The change in volume is detected by using a respirometer over a given period of time. The data are given below. Energy Unit test #27 The rate of oxygen consumption in germinating pea seeds at 26ºC is A 0.05 mL / min B 0.25 mL / min C 0.50 mL / min D 0.75 mL / min E 1.00 mL / min

A

The reactions of glycolysis occur in the A cytosol B nucleus C matrix of the mitochondrion D membranes of the mitochondrion E stroma of the chloroplast

A

Transcription review questions #1 . . . . glutamine-glutamine-glutamine . . . . . . serine-serine-serine . . . Which of the following messenger RNA sequences could code for both of the two amino acid sequences above, simply by a shift in the reading frame? . . . AGCAGCAGCAGC . . . A . . . AGUAGUAGUAGU . . . B . . . CAACAACAACAA . . . C . . .GCUGCUGCUGCU . . . D . . . GCAAGCGCAAGC . . .

A

Translation practice questions # 6 Molecular biologists are studying the processes of transcription and translation and have found that they are very similar in prokaryotes and eukaryotes, as summarized in Table 1. Based on the information in Table 1, which of the following best predicts a key difference in prokaryotes and eukaryotes with regard to transcription and translation? A The two processes will occur simultaneously in prokaryotes but not eukaryotes. B Prokaryotic mRNA is shorter than eukaryotic mRNA. C Eukaryotic mRNA contains more coding regions than prokaryotic DNA. D The processing of mRNA by eukaryotes is required for the mRNA to leave the nucleus.

A

Two nutrient solutions are maintained at the same pH. Actively respiring mitochondria are isolated and placed into each of the two solutions. Oxygen gas is bubbled into one solution. The other solution is depleted of available oxygen. Which of the following best explains why ATP production is greater in the tube with oxygen than in the tube without oxygen? A The rate of proton pumping across the inner mitochondrial membrane is lower in the sample without oxygen. B Electron transport is reduced in the absence of a plasma membrane. C In the absence of oxygen, oxidative phosphorylation produces more ATP than does fermentation. D In the presence of oxygen, glycolysis produces more ATP than in the absence of oxygen.

A

Type 1 diabetes results from the destruction of insulin-producing cells in the pancreas. Individuals with type 1 diabetes produce insufficient amounts of insulin, a hormone that regulates the concentration of glucose in the blood. Which of the following best explains how treatment with a drug that stimulates the production of insulin receptors on target cells will affect the insulin signaling pathway in an individual with type 1 diabetes? A The drug will have little or no effect on the signaling pathway because the receptors will not be activated in the absence of insulin. B The drug will have little or no effect on the signaling pathway because insulin receptors will not be allowed to enter the cells. C The drug will restore the function of the signaling pathway because insulin levels will return to normal. D The drug will restore the function of the signaling pathway because nonpancreatic cells will begin to produce insulin receptors.

A

Unit 5 practice questions #22 In a strain of tomato plants, short plant height and small fruit size are traits that display autosomal recessive patterns of inheritance. To investigate whether the traits segregate independently, researchers cross a pure-breeding line of tall tomato plants that have large fruits with a pure-breeding line of short tomato plants that have small fruits. The researchers observe that all the plants in the F1 generation are tall and have large fruits. The researchers cross the F1 plants with one another to generate an F2generation. The researchers record observations for the F2 generation and will use the data to perform a chi-square goodness-of-fit test for a model of independent assortment. The setup for the chi-square goodness-of-fit test is shown in Table 1. Table 1. Observed and expected counts of tomato plants in the F2 generation PhenotypeObservedExpectedTall plant with large fruits10490Tall plant with small fruits1730Short plant with large fruits2630Short plant with small fruits1310 The researchers choose a significance level of p=0.05. Which of the following best completes the chi-square goodness-of-fit test? A The calculated chi-square value is 9.24, and the critical value is 7.82. The null hypothesis of independent assortment can be rejected. B The calculated chi-square value is 9.24, and the critical value is 9.49. The null hypothesis of independent assortment cannot be rejected. C The calculated chi-square value is 13.13, and the critical value is 7.82. The null hypothesis of independent assortment can be rejected. D The calculated chi-square value is 13.13, and the critical value is 9.49. The null hypothesis of independent assortment cannot be rejected.

A

Which of the following best describes the role of mitosis in the cell cycle? A Distributing replicated chromosomes to daughter nuclei B Dividing the cytoplasm to form four gametes C Producing organelles and replicating chromosomes D Exchanging genetic material between homologous chromosomes

A

Which of the following statements concerning a gene is correct? A A gene can code for a specific protein. B A gene can exist in alternate forms called introns. C A gene undergoes crossing-over during DNA replication. D A gene that is very similar in sequence in a human and in a bacterium is probably a recent mutation. E A gene that is expressed in every offspring of every generation is recessive.

A

Which of the following statements is true regarding the movement of substances across cell membranes? A Ions are unable to move through the phospholipid bilayer because the nonpolar tail regions of the phospholipids are hydrophobic. B Ions are able to move through the phospholipid bilayer because the polar head regions of the phospholipids are charged. C Water is able to move through the phospholipid bilayer because the nonpolar tail regions of the phospholipids are charged. D Water is unable to move through the phospholipid bilayer because the polar head regions of the phospholipids are charged.

A

Within the cell, many chemical reactions that, by themselves, require energy input (have a positive free-energy change) can occur because the reactions A may be coupled to the hydrolysis of ATP B take place very slowly C take place when the cells are at unusually high temperatures D are catalyzed by enzymes E are aided by various metal ions that act as catalysts

A

unit 5 practice questions #16 A student crosses a pure-breeding line of red-flowered poinsettias with a pure-breeding line of white-flowered poinsettias. The student observes that all the plants in the F1 generation have pink flowers. The student then crosses the F1 plants with one another and records observations about the plants in the F2 generation. The student will use the F2 data to perform a chi-square goodness-of-fit test for a model of incomplete dominance. The setup for the student's chi-square goodness-of-fit test is presented in Table 1. Table 1. A chi-square goodness-of-fit test for incomplete dominance PhenotypeObservedExpectedRed flowers6456Pink flowers107112White flowers5356 The critical value for a chi-square test with a significance level of p=0.05 and 2 degrees of freedom is 5.99. Which of the following statements best completes the student's chi-square goodness-of-fit test? A The calculated chi-square value is 1.53, and the null hypothesis cannot be rejected. B The calculated chi-square value is 1.53, and the null hypothesis can be rejected. C The calculated chi-square value is 98, and the null hypothesis cannot be rejected. D The calculated chi-square value is 98, and the null hypothesis can be rejected.

A

(A) DNA ligase (B) DNA polymerase (C) RNA polymerase (D) Restriction enzyme (E) Reverse transcriptase Enzyme used to position nucleotides during DNA replication A DNA ligase B DNA polymerase C RNA polymerase D Restriction enzyme E Reverse transcriptase

B

(A) Glysolysis (B) Krebs cycle (citric acid cycle) (C) Calvin cycle (light-independent reactions of photosynthesis) (D) Light-dependent reactions of photosynthesis (E) Chemiosmosis Process in which CO2, is released as a by-product of oxidation-reduction reactions A B C D E

B

(A) Transcription(B) Translation(C) Transformation(D) Replication(E) Reverse transcription Process in which a protein is assembled at a ribosome A B C D E

B

9For the following questions: Graphs I-IV depict the effect of pH on the activity of four different hydrolytic enzymes. Energy unit test # 7 Enzymes with their highest activity at an alkaline (basic) pH are represented by which of the following graphs? A I only B II only C III only D I and III only E I and IV only

B

A blue-flowered African violet of unknown ancestry self-pollinated and produced 50 seeds. These seeds germinate and grow into flowering plants. Of these plants, 36 produce blue flowers and 14 produce pink flowers. What is the best explanation for the pink-flowered offspring? A Blue flowers are incompletely dominant to pink flowers. B Pink flower color is a trait recessive to blue flower color. C Pink flower color is the result of somatic mutations in the flower color gene. D A previous generation of the blue-flowered parent must have included 50 percent pinkflowered plants.

B

A couple has 5 children, all sons. If the woman gives birth to a sixth child, what is the probability that the sixth child will be a son? A 5/6 B 1/2 C 1/5 D 1/6 E 1/64

B

A genetic counselor is consulted by a young man who is worried about developing Huntington's disease, an inherited disorder caused by a dominant allele of a single gene. The young man explains that his cousin was recently diagnosed with Huntington's disease, and the news has caused him to consider his own risk of developing the disorder. Which of the following questions will best help the genetic counselor to evaluate the risk of the young man developing Huntington's disease and transmitting it to his children? A Were you and your cousin born in the same geographical area? B Were your parents or grandparents ever diagnosed with Huntington's disease? C Were you in physical contact with a person diagnosed with Huntington's disease? D Were you ever exposed to substances that are suspected of being mutagens?

B

A researcher claims that different metabolic pathways allow bacteria to use different molecules as sources of matter and energy. Which of the following statements best helps justify the researcher's claim by providing a relevant example? A Rhizobia bacteria form close associations with the roots of bean plants. B E. coli bacteria reproduce in liquid media containing either glucose or galactose. C The antibiotic rifampicin inhibits the growth of some bacterial strains but not of others. D Some viruses that infect bacteria reproduce by either the lysogenic cycle or the lytic cycle.

B

A researcher claims that the synthesis of ATP from ADP and inorganic phosphate (Pi) is essential to cellular function. Which of the following statements best helps justify the researcher's claim? A ADPADP is a small molecule that some cells release into their environment as a way of communicating with other cells. B ATPATP hydrolysis is an energy-releasing reaction that is often coupled with reactions that require an input of energy. C Inorganic phosphate (Pi)(Pi) is a substance that cells typically acquire from their environment. D ATPATP synthase is a mitochondrial enzyme that catalyzes the conversion of ADPADP and PiPi to ATPATP.

B

A researcher examining a root tip observes a plant cell with condensed sister chromatids, kinetochores with attached microtubules, and individual chromosomes that are aligned at the equatorial plate of the cell. Which of the following best describes what the next process will be in the cell? A Homologous chromosomes (each with two sister chromatids) will move toward opposite poles of the cell. B Paired chromatids will separate, and the new daughter chromosomes will move toward opposite poles of the cell. C The nuclear envelope will break down, and the spindle will begin to form. D The chromatin will decondense, and the daughter cell will enter interphase.

B

A scientist claims that Elysia chlorotica, a species of sea slug, is capable of photosynthesis. Which of the following observations provides the best evidence to support the claim? A Elysia chlorotica will die if not exposed to light. B Elysia chlorotica grows when exposed to light in the absence of other food sources. C Elysia chlorotica grows faster when exposed to light than when placed in the dark. D Elysia chlorotica grows in the dark when food sources are available.

B

A student analyzed a viral genome and found that the genome had the following nucleotide composition. • 28% adenine • 20% thymine • 35% cytosine • 17% guanine Which of the following best describes the structure of the viral genome? A Double-stranded DNA B Single-stranded DNA C Double-stranded RNA D Single-stranded RNA Related Content & Skills Topic1.6 SkillSkill 6.B

B

Bacteriophages are viruses that infect bacteria. In an experiment, bacteriophages were labeled with either radioactive phosphorus or radioactive sulfur. The labeled bacteriophages were incubated with bacteria for a brief amount of time and then removed. The infected bacteria cells were found to contain significant amounts of radioactive phosphorus but not radioactive sulfur. Based on the results of the experiment, which of the following types of molecules did the bacteriophages most likely inject into the bacteria cells? A Simple carbohydrate B Amino acid C DNA D Polypeptide Related Content & Skills Topic1.2 SkillSkill 6.B

B

Carbohydrate-synthesizing reactions of photosynthesis directly require A light B products of the light reactions C darkness D O2 and H2O E chlorophyll and CO2

B

Cell communication unit test #2 The p53 protein regulates a cellular response to DNA damage. Based on the diagram above, which of the following best describes the role of p53 in the response to DNA damage? A Phosphorylated p53 binds to DNA and repairs the damage. B Phosphorylated p53 stimulates the transcription of p21, and the resulting p21 protein suppresses cell division until the DNA damage is repaired. C Phosphorylated p53 binds cyclin−Cdk complexes, and the resulting protein complex repairs the DNA damage. D Phosphorylated p53 activates p21 proteins, and the p21 proteins in turn repair the DNA damage.

B

Cell communication unit test #23 The model shown in the figure represents the role of two hormones, calcitonin and parathyroid hormone (PTH), in maintaining normal blood calcium levels in humans. If a dietary change results in an increase in blood calcium concentration above normal levels, which of the following is the most likely effect on calcium homeostasis? A Calcitonin levels will decline, thus stimulating the release of PTH. B Calcitonin levels will rise, thus promoting the deposit of calcium into bones. C PTH levels will decline, thus stimulating the loss of calcium from bones. D PTH levels will increase, thus preventing the release of calcitonin.

B

Cell signaling quiz # 15 Glucocorticoids are steroid hormones that control cellular responses through several different signaling pathways. One of the signaling pathways involves the glucocorticoid receptor, an intracellular protein that is activated by binding to a glucocorticoid molecule. A simplified model of the glucocorticoid receptor signaling pathway is represented in Figure 1. Figure 1. A simplified model of the glucocorticoid receptor signaling pathway Which of the following statements best predicts the effect of a mutation that results in a loss of the glucocorticoid receptor's ligand binding function? A The transduction of the glucocorticoid signal across the plasma membrane will be blocked. B The glucocorticoid receptor will remain associated with the accessory proteins. C The rate of diffusion of glucocorticoid molecules into the cell will increase. D The concentration of glucocorticoid receptors inside the nucleus will increase.

B

Cell signaling quiz # 8 The diagram above represents a model of signal transduction pathways (I and II) in a cell that is targeted by two different hormones (H1 and H2). The components of the signal transduction pathways are identified in the figure legend. Each cellular molecule in both pathways can exist in an inactive or active form. When the components in pathway I are sequentially activated, the TAP molecules promote cell division. When the components in pathway II are sequentially activated, downstream signaling by the G protein is inhibited. Based on the model, which of the following mutations is most likely to result in a cell that will generate a cancerous tumor? A A mutation in the gene encoding PP that results in a nonfunctional protein B A mutation in the gene encoding G-PIP that results in a nonfunctional protein C A mutation in the gene encoding R1 so that it is inactive even in the presence of H1 D A mutation in the gene encoding R2 so that it is active even in the absence of H2

B

Coenzyme that transfers electrons from the Krebs cycle to the mitochondrial electron-transport chain at a lower energy level than that of electrons entering at the beginning of the chain A Cytochrome B FADH2 C NAD+ D NADP+ E Oxygen (O2)

B

Directions: Each group of questions below concerns an experimental or laboratory situation or data. In each case, first study the description of the situation or data. Then choose the one best answer to each question following it. CR quiz #35 A tissue culture of vertebrate muscle was provided with a constant excess supply of glucose under anaerobic conditions starting at time zero and the amounts of pyruvic acid and ATP produced were measured. The solid line in the graph above represents the pyruvic acid produced in moles per liter per minute. ATP levels were also found to be highest at points A and C, lowest at B and D. A second culture was set up under the same conditions, except that substance X was added, and the results are indicated by the dotted line. Which of the following is most likely to result if oxygen is added to the tissue culture? A Lactic acid formation will increase. B For each glucose molecule consumed, more ATP will be formed. C The levels of ATP produced will decrease. D Ethyl alcohol will be produced. E No change in the production of pyruvic acid will be observed.

B

Directions: Each group of questions below concerns an experimental or laboratory situation or data. In each case, first study the description of the situation or data. Then choose the one best answer to each question following it. Energy Unit Test #13 A tissue culture of vertebrate muscle was provided with a constant excess supply of glucose under anaerobic conditions starting at time zero and the amounts of pyruvic acid and ATP produced were measured. The solid line in the graph above represents the pyruvic acid produced in moles per liter per minute. ATP levels were also found to be highest at points A and C, lowest at B and D. A second culture was set up under the same conditions, except that substance X was added, and the results are indicated by the dotted line. It is most reasonable to hypothesize that, in the breakdown of glucose, substance X is A an activator B an inhibitor C a substrate D a coenzyme E a cofactor

B

During mitosis, which of the following normally occurs? A Homologous chromosomes pair. B Replicated chromosomes line up on the equatorial plate. C Tetrads form. D Unreplicated chromosomes become oriented in the center of the cell. E Maternal and paternal chromatids pair.

B

During respiration, most ATP is formed as a direct result of the net movement of A potassium against a concentration gradient B protons down a concentration gradient C electrons against a concentration gradient D electrons through a channel E sodium ions into the cell

B

Feedback Practice Questions #2 Figure 1 is a proposed model of the feedback system controlling erythrocyte (red blood cell) production. Figure 1. Model of erythrocyte production control Air is less dense at very high elevations, so less oxygen is available than in the denser air at sea level. Based on the model in Figure 1, if a person travels from sea level to a high elevation location, which of the following correctly predicts the response to the decreased blood oxygen level? A More erythropoietin will be secreted from the kidneys, decreasing production of erythrocytes. B More erythropoietin will be secreted from the kidneys, increasing production of erythrocytes. C Less erythropoietin will be secreted from the kidneys, decreasing production of erythrocytes. D Less erythropoietin will be secreted from the kidneys, increasing production of erythrocytes.

B

Feedback Practice questions #3 The coagulation cascade controls blood clot formation in response to blood vessel injury. Thrombin is an enzyme that plays a key role in regulating the coagulation cascade. A simplified model of thrombin's role in regulating the coagulation cascade is represented in Figure 1. Figure 1. A simplified model of thrombin's role in regulating the coagulation cascade Argatroban is a competitive inhibitor of thrombin. Which of the following effects on the coagulation cascade is most likely to result from inhibiting thrombin activity with argatroban? A The activation of clotting factors will be blocked. B The rate of fibrin formation will decrease. C Thrombin will be converted to prothrombin. D The rate of blood clot formation will increase.

B

Figure 1 represents a nucleic acid fragment that is made up of four nucleotides linked together in a chain. Figure 1. Nucleic acid fragment Which of the following characteristics of Figure 1 best shows that the fragment is RNA and not DNA? Chem of life 8 A The 5′5′ to 3′3′ orientation of the nucleotide chain B The identity of each nitrogenous base C The charges on the phosphate groups D The type of bond linking the nucleotides together

B

Figure 1 shows a model of a signal transduction cascade, initiated by the binding of a ligand to the transmembrane receptor proteinA. Figure 1. Model of signal transduction cascade incorporating protein A Cell signaling quiz review # 9 A DNA mutation changes the shape of the extracellular domain of transmembrane receptor protein A produced by the cell. Which of the following predictions is the most likely consequence of the mutation? A Production of activated molecule 1 will stop, but production of activated molecules 2 and 3 will continue. B The molecule that normally binds to protein AA will no longer attach, deactivating the cellular response. C The molecule that normally binds to protein AA will not enter the cell, thus no cellular response will occur. D Since protein AA is embedded in the membrane, the mutation will be silent and not affect the cellular response.

B

Figure 1. Nucleic acid segments DNA and RNA are nucleic acids that can store biological information based on the sequence of their nucleotide monomers. Figure 1 shows a short segment of each of the two types of nucleic acids. Which of the following best describes a structural difference between DNA and RNA? Test redo quiz 11 A DNADNA contains four types of nitrogenous bases, whereas RNARNA contains only two types of nitrogenous bases. B The backbone of DNADNA contains deoxyribose, whereas the backbone of RNARNA contains ribose. C A DNADNA molecule is composed of two parallel strands with the same 5′5′ to 3′3′ directionality, whereas anRNARNA molecule is composed of only one 5′5′ to 3′3′ strand. D Phosphate groups provide rigidity to DNADNA, but RNARNA is flexible and contains no phosphate groups.

B

For following group of questions first study the description of the situation and diagrams and then choose the one best answer to each question following it and fill in the corresponding oval on the answer sheet. A student studied the effects of light intensity on oxygen production in green algae. The algae were suspended in water inside a sealed glass jar, and the jar was placed into a constant-temperature, lightproof box containing a light source. A probe was inserted into the jar to record the concentration of oxygen. The probe was connected to a recording device. The setup is shown below. The student decreased the intensity of the light hourly and recorded the corresponding changes in oxygen concentration. The graph below shows the results from the recording device. Based on the data shown, changes in the light intensity resulted in changes in the rate of which of the following processes? Photosynthesis quiz #5 A Excretion B Photosynthesis C Respiration D Translation E Transcription

B

Hereditary unit test #18 For following group of questions first study the description of the situation and diagram and then choose the one best answer to each question following it and fill in the corresponding oval on the answer sheet. The pedigree of a family with a history of a particular genetic disease is shown below. Squares represent males and circles represent females. Shaded symbols represent those who have the disease. If Individual 2 were to marry a woman with no family history of the disease, which of the following would most likely be true of their children? A All of the children would have the disease. B None of the children would have the disease. C Only the sons would have the disease. D All of the sons would be carriers of the disease. E None of the daughters would be carriers of the disease.

B

In mammals, an increase in the concentration of sodium in the blood triggers the release of antidiuretic hormone (ADH) from the pituitary gland. As the concentration of sodium in the blood returns to previous levels, the release of ADH from the pituitary gland is reduced. Based on the information presented, which of the following describes the most likely role of ADH in maintaining blood osmolarity? A ADH promotes an increase in the movement of sodium into the bloodstream. B ADH promotes an increase in the movement of water into the bloodstream. C ADH promotes an increase in the excretion of water from the body. D ADH promotes an increase in the secretion of additional ADH from the pituitary gland.

B

In sheep, eye color is controlled by a single gene with two alleles. When a homozygous brown-eyed sheep is crossed with a homozygous green-eyed sheep, blue-eyed offspring are produced. If the blue-eyed sheep are mated with each other, what percent of their offspring will most likely have brown eyes? A 0% B 25% C 50% D 75% E 100%

B

Mutations in DNA are usually caused by chemical or radiation damage to DNA molecules, followed by imperfect repair of the damage. Immediately after this kind of imperfect repair, there may be a mismatched base pair in the DNA. The illustration below shows an example of a mismatch, with the relevant pair bases in bold. Which of the following best represents the DNA of the two daughter cells produced when a bacterial cell with this particular mismatch replicates its DNA and divides? Test Redo unit one question 4 A B C D

B

Researchers compared similar proteins from related organisms in different habitats. They found that the proteins from organisms living in harsh environments had a greater number of cysteine amino acids than did proteins from organisms not living in harsh environments. The structure of cysteine is shown. Bonds can form between the sulfur atom of different cysteine amino acids (S-Sbonds). Figure 1. Chemical structure of cysteine Which of the following best describes the effect of a greater number of cysteine amino acids on the stability of the proteins?Chem of Life question 5 A The change has no effect on the stability of the protein because only one type of amino acid is involved. B The change leads to increased protein stability because of an increased number of S-SS-S bonds in the tertiary structure of the proteins. C The change leads to decreased protein stability because of an increased number of S-SS-S bonds in the tertiary structure of the proteins. D The change leads to increased protein stability only when the added cysteine amino acids are next to other cysteine amino acids in the primary structure.

B

Simple diffusion and facilitated diffusion are related in that both A require protein carriers B depend on a concentration gradient C occur via contractions of cytoskeletal elements attached to membrane proteins D are endergonic processes and thus require the hydrolysis of ATP E occur in eukaryotic cells but not in prokaryotic cells

B

Students investigated the effect of light on the carbon cycle in aquatic ecosystems by performing the controlled experiment summarized below. The students placed equal amounts of water (pH 7.0) from a large aquarium in glass beakers. The students transferred aquatic plants from the aquarium to several of the beakers, and then they placed equal numbers of the beakers in the light or the dark (Figure 1: groups I and II). Similarly, the students transferred goldfish from the same aquarium to other beakers, and then they placed equal numbers of those beakers in the light or dark (Figure 1: groups III and IV). Finally, the students placed an equal number of beakers containing water only in the light or dark (Figure 1: groups V and VI). After exposing the samples to light or dark for one hour, the students recorded the pH of the water in each beaker. Carbon dioxide dissolved in water will lower the pH of an aqueous solution. In the experiment, the students used changes in pH to monitor changes in the amount of carbon dioxide in the water. For each treatment group, the students calculated the mean pH and standard error, as documented in the table below. Which of the following graphs is the most appropriate representation of the experimental results documented in the table? Chem of life #24 A B C D

B

The O2 released during photosynthesis comes from A CO2 B H2O C NADPH D RuBP (RuDP) E C6H12O6

B

The carbon 'that makes up organic molecules in plants is derived directly from A combustion of fuels B carbon fixed in photosynthesis C carbon dioxide produced in respiration D carbon in the lithosphere E coal mines

B

The chemical reaction for photosynthesis is 6 CO2 + 12 H2O + light energy → C6H12O6 + 6 O2 + 6 H2O If the input water is labeled with a radioactive isotope of oxygen, 18O, then the oxygen gas released as the reaction proceeds is also labeled with 18O. Which of the following is the most likely explanation? A During the light reactions of photosynthesis, water is split, the hydrogen atoms combine with the CO2, and oxygen gas is released. B During the light reactions of photosynthesis, water is split, removing electrons and protons, and oxygen gas is released. C During the Calvin cycle, water is split, regenerating NADPH from NADP+, and oxygen gas is released. D During the Calvin cycle, water is split, the hydrogen atoms are added to intermediates of sugar synthesis, and oxygen gas is released.

B

The chemical reaction for photosynthesis is 6CO2 + 12H2O + light energy →C6H12O6 + 6O2 + 6H2O If the input water is labeled with a radioactive isotope of oxygen, 18O, then the oxygen gas released as the reaction proceeds is also labeled with 18O. Which of the following is the most likely explanation? A During the light reactions of photosynthesis, water is split, the hydrogen atoms combine with the CO2, and oxygen gas is released. B During the light reactions of photosynthesis, water is split, removing electrons and protons, and oxygen gas is released. C During the Calvin cycle, water is split, regenerating NADPH from NADP+, and oxygen gas is released. D During the Calvin cycle, water is split, the hydrogen atoms are added to intermediates of sugar synthesis, and oxygen gas is released.

B

The endocrine system incorporates feedback mechanisms that maintain homeostasis. Which of the following demonstrates negative feedback by the endocrine system? A During labor, the fetus exerts pressure on the uterine wall, inducing the production of oxytocin, which stimulates uterine wall contraction. The contractions cause the fetus to further push on the wall, increasing the production of oxytocin. B After a meal, blood glucose levels become elevated, stimulating beta cells of the pancreas to release insulin into the blood. Excess glucose is then converted to glycogen in the liver, reducing blood glucose levels. C At high elevation, atmospheric oxygen is more scarce. In response to signals that oxygen is low, the brain decreases an individual's rate of respiration to compensate for the difference. D A transcription factor binds to the regulatory region of a gene, blocking the binding of another transcription factor required for expression.

B

The epinephrine signaling pathway plays a role in regulating glucose homeostasis in muscle cells. The signaling pathway is activated by the binding of epinephrine to the beta-2 adrenergic receptor. A simplified model of the epinephrine signaling pathway is represented in Figure 1. Cell signaling quiz review # 4 Figure 1. A simplified model of the epinephrine signaling pathway in muscle cells Based on Figure 1, which of the following statements best describes the epinephrine signaling pathway? A It involves the opening and closing of ion channels. B In involves enzymes activating other enzymes. C It involves changes in the expression of target genes. D It involves protons moving down a concentration gradient.

B

The figure above shows an organelle typically found in eukaryotic cells. Which of the following best describes the function of the double membrane system of this organelle? CR quiz #26 A The outer membrane allows the transport of all molecules into the intermembrane space, while the inner membrane serves as the regulatory boundary. B The inner membrane has specialized proteins that create a hydrogen ion concentration gradient between the intermembrane space and the matrix. C The outer membrane contains transport proteins that establish a sodium ion concentration gradient used for ATP production, while the inner membrane contains transport proteins that establish a hydrogen ion concentration gradient used for glucose production. D The toxins and wastes entering a cell cross the outer membrane and are detoxified by digestive enzymes stored within the intermembrane space.

B

The following questions refer to an experiment in which a dialysis-tubing bag is filled with a mixture of 3% starch and 3% glucose and placed in a beaker of distilled water, as shown below. After 3 hours, glucose can be detected in the water outside the dialysis-tubing bag, but starch cannot. From the initial conditions and results described, which of the following is a logical conclusion? A The initial concentration of glucose in the bag is higher than the initial concentration of starch in the bag. B The pores of the bag are larger than the glucose molecules but smaller than the starch molecules. C The bag is not selectively permeable. D A net movement of water into the beaker has occurred. E The molarity of the solution in the bag and the molarity of the solution in the surrounding beaker are the same.

B

The mechanism of action of many common medications involves interfering with the normal pathways that cells use to respond to hormone signals. Which of the following best describes a drug interaction that directly interferes with a signal transduction pathway? A A medication causes the cell to absorb more of a particular mineral, eventually poisoning the cell. B A medication enters the target cell and inhibits an enzyme that normally synthesizes a second messenger. C A medication enters the target cell's nucleus and acts as a mutagen. D A medication interrupts the transcription of ribosomal RNA genes.

B

The salinity of a small inland lake has recently started to increase. Researchers are planning to study the lake over several decades to investigate how freshwater organisms survive significant changes in their natural habitat. Which of the following physiological mechanisms will the researchers most likely observe among the surviving organisms in the lake? A Prokaryotic organisms will use various mechanisms to counteract swelling of cells as a result of increased water uptake. B Single-celled organisms will use various mechanisms to counteract the increased flow of water from cells to the environment. C Eukaryotic organisms will use various mechanisms to counteract the diffusion of positively charged ions across the cell membrane. D Multicellular organisms will use various mechanisms to counteract the loss of cell adhesion as a result of calcium deficiencies.

B

This group of questions refers to the probabilities below. Assume that the alleles referred to all assort independently. (A) 0 (B) 1/16 (C) 1/4 (D) 1/2 (E) 3/4 Probability that the genotype ccdd will be produced by the parents CcDd x CcDd A 0 B 1/16 C 1/4 D 1/2 E 3/4

B

To test the hypothesis that a particular plant synthesizes storage lipids by using glyceraldehyde 3-phosphate (G3P) from photosynthesis, a researcher plans to use radiolabeled precursors to track the molecules through the biosynthetic pathway. Which of the following radiolabeled precursors is most appropriate for the researcher to use? A 15N-labeled N2, because atmospheric nitrogen is fixed to amino acids by photosynthesis B 14C-labeled CO2, because atmospheric carbon is fixed to carbohydrates by photosynthesis C 35S-labeled methionine, because amino acids are incorporated into lipids during photosynthesis D 32P-labeled phosphate, because lipids are stored in plants as phospholipids

B

Two groups of students attempted to re-create the primitive atmospheric conditions of early Earth using the apparatus represented below. Each group ran the experiment with different gas mixtures in the apparatus. Which of the following statements best justifies the claim that the conditions in at least one of the experiments could generate the molecular building blocks essential for life? A The carbon dioxide gas and water vapor in experiment 1 could react spontaneously to produce the phospholipids required by the first life-forms. B The nitrogen gas in experiment 1 could provide the elemental nitrogen required for the formation of amino acids. C The sulfur dioxide gas in experiment 2 could donate the excited electrons required to drive the process of photosynthesis. D The methane gas in experiment 2 could act as the electron acceptor required to complete the process of cellular respiration

B

Unit 5 Practice questions #7 The Russian blue is a rare breed of cat that is susceptible to developing cataracts on the eyes. Scientists hypothesize that this condition is inherited as a result of a mutation. Figure 1 shows a pedigree obtained in a study of cats owned by members of the Russian Blue Club in Sweden. Figure 1. Pedigree of Russian blue cats owned by Russian Blue Club Based on the inheritance pattern shown in Figure 1, which of the following best predicts the nature of the original mutation? A A recessive mutation on the X chromosome B A recessive mutation on a somatic chromosome C A dominant mutation on the X chromosome D A dominant mutation on a somatic chromosome

B

Unit 5 practice questions #13 An experiment was performed to determine the mode of inheritance of two mouse genes, one for fur color and one for fur length. It is known that black fur (B) is dominant over white fur (b) and that long fur (L) is dominant over short fur (l). To determine how the genes are inherited, a cross was performed between two true-breeding mice, one with long black fur and one with short white fur. Their progeny, the F1 generation, all had long black fur. Five F1 male-female pairs were then crossed with one another. The F2 generation phenotypes for each cross are shown in Table·1. Table 1. Number of F2 generation phenotypes for five crosses PhenotypeCross 1Cross 2Cross 3Cross 4Cross 5Long black fur65567Long white fur11022Short black fur02111Short white fur23322 Which of the following is the mean number per cross of F2 generation offspring that are the result of crossing over? A 1 B 2.2 C 2.4 D 5.8

B

Water and ammonia interact to form hydrogen bonds, as represented in the figure. Which statement best helps explain the formation of the hydrogen bond represented in the figure? A The oxygen has a partial positive charge, and the nitrogen has a partial negative charge. B The nitrogen has a partial negative charge, and the hydrogen attached to the oxygen has a partial positive charge. C The hydrogen attached to the oxygen has a partial negative charge, and the nitrogen also has a partial negative charge. D The nitrogen has a partial positive charge, and the hydrogen attached to the oxygen also has a partial positive charge.

B

Which metabolic process is common to both aerobic cellular respiration and alcoholic fermentation? A Krebs cycle B Glycolysis C Electron transport chain D Conversion of pyruvic acid to acetyl CoA E Production of a proton gradient

B

Which of the following best describes an event during step 2 in the simplified model above? A A new RNA molecule is synthesized using a DNA template. B A new polypeptide is synthesized using an RNA template. C Thymine nucleotides in an RNA molecule are replaced with uracil nucleotides. D Which of the following normally leads to the production of functional messenger RNA in eukaryotic cells? A A decrease in the rate of ribosome synthesis B The removal of portions of RNA known as intervening sequences (introns) C A decrease in RNA polymerase activity D The replication of new messenger RNA molecules from existing messenger RNA molecules E The formation of peptide bonds between adjacent nucleotides

B

Which of the following best describes how amino acids affect the tertiary structure of a protein? A The number of amino acids determines the tertiary structure of the protein. B The interactions of the different RR-groups with other RR-groups and with their environment determine the tertiary structure of the protein. C The RR-group of the last amino acid that is added to a growing polypeptide chain determines the next amino acid that is added to the chain. D The sequence of the amino acids in the polypeptide chain determines the protein's primary structure but has no effect on its tertiary structure.

B

Which of the following best describes the structures of carbohydrates? A They only occur as disaccharides. B They occur as monomers, chains of monomers, and branched structures. C They only occur as long and branched structures. D They occur as chains of monomers that hydrogen bond with complementary chains of monomers.

B

Which of the following best explains what process is represented in Figure 1 ? A New DNADNA strands are being synthesized in the 3'3′ to 5'5′ direction from their DNADNA templates. B New DNADNA strands are being synthesized in the 5'5′ to 3'3′ direction from their DNADNA templates. C A new RNARNA strand is being synthesized in the 3'3′ to 5'5′ end from its DNADNA template. D Two new RNARNA strands are being synthesized in both directions from their DNADNA templates.

B

Which of the following can be used to determine the rate of enzyme-catalyzed reactions? A Rate of disappearance of the enzyme B Rate of disappearance of the substrate C Rate of disappearance of the product D Change in volume of the solution E Increase in activation energy

B

Which of the following components of the cell membrane is responsible for active transport? A Phospholipid B Protein C Lipid D Phosphate E Cholesterol

B

Which of the following is true of mitosis? A It is also known as cytokinesis. B It maintains the same chromosome number in the daughter cells as in the parent cell. C It is the last phase of interphase. D It regulates the transfer of genetic information from one daughter cell to another. E It moves homologous chromosomes to opposite poles.

B

Which of the following questions is most relevant to understanding the Calvin cycle? A How does chlorophyll capture light? B How is ATP used in the formation of 3-carbon carbohydrates? C How is NADP+ reduced to NADPH? D How is ATP produced in chemiosmosis?

B

Which of the following questions will best direct an investigation of the mechanism of ATP synthase? CR topic questions #1 A What is the source of the inorganic phosphate that is used to generate ATP from ADP? B Is the phosphorylation of ADP by ATP synthase dependent on the formation of a proton gradient? C Can ATP synthase use the energy released by phosphorylation of ADP to pump protons against a concentration gradient? D Can oxidative phosphorylation be uncoupled from the electron transport chain?

B

transcription review questions #3 A model that represents a process occurring in a cell of a particular organism is shown in Figure 1. Figure 1. Process occurring in a cell Which of the following correctly explains the process shown in Figure 1 ? DNADNA replication is occurring because replication is semi-conservative and the new strand is a copy of the template strand. A Initiation of transcription is occurring because a strand of RNARNA is being produced from a DNADNA template strand. B Translation is occurring because the two strands have separated and a new strand is being produced. C Alternative splicing of mRNAmRNA is occurring because the mRNAmRNA strand is being synthesized from only one strand of DNADNA.

B

(A) DNA ligase (B) DNA polymerase (C) RNA polymerase (D) Restriction enzyme (E) Reverse transcriptase Enzyme used in the synthesis of mRNA A DNA ligase B DNA polymerase C RNA polymerase D Restriction enzyme E Reverse transcriptase

C

(A) Glysolysis (B) Krebs cycle (citric acid cycle) (C) Calvin cycle (light-independent reactions of photosynthesis) (D) Light-dependent reactions of photosynthesis (E) Chemiosmosis Process in which carbon from CO2 is incorporated into organic molecules A B C D E

C

A feature of organic compounds NOT found in inorganic compounds is the presence of A ionizing chemical groups B electrons C carbon atoms covalently bonded to each other D oxygen E hydrogen bonds

C

A researcher claims that only a portion of the light energy captured by green plants is available for growth and repair. Which of the following observations best helps justify the researcher's claim? A Light-capturing pigment molecules in green plants absorb red, blue, and violet light but reflect green light. B The energy of a photon of light is proportional to its frequency and inversely proportional to its wavelength. C As light energy is converted to chemical energy by metabolic processes, some of the energy is lost as heat. D Captured energy is stored in the molecular bonds of organic molecules, including simple sugars and starch.

C

A sample of human blood was placed in a test tube containing a physiological saline solution (0.9% sodium chloride). This type of solution is often used intravenously to quickly rehydrate patients. A drop of the blood from the test tube was placed on a slide and red blood cells (RBCs) were observed under a microscope. Three possible outcomes are diagrammed below. Cells quiz #3 #15 Which of the following best predicts which diagrammed microscope view the laboratory worker would see and best explains why? A View 1 because RBC membranes are freely permeable to water B View 2 because the RBCs use energy to allow sodium entry and to pump water out C View 2 because the rate of water movement into the RBCs equals the rate of water movement out of the cells D View 3 because the sodium-potassium pumps in the RBC membranes use energy to keep the sodium out but allow water to freely flow into the cells

C

A student placed 20 tobacco seeds of the same species on moist paper towels in each of two petri dishes. Dish A was wrapped completely in an opaque cover to exclude all light. Dish B was not wrapped. The dishes were placed equidistant from a light source set to a cycle of 14 hours of light and 10 hours of dark. All other conditions were the same for both dishes. The dishes were examined after 7 days, and the opaque cover was permanently removed from dish A. Both dishes were returned to the light and examined again at 14 days. The following data were obtained. Photosynthesis quiz #3 Additional observations were made on day 21, and no yellow-leaved seedlings were found alive in either dish. This is most likely because A yellow-leaved seedlings were unable to absorb water from the paper towels B taller green-leaved seedlings blocked the light and prevented photosynthesis C yellow-leaved seedlings were unable to convert light energy to chemical energy D a higher rate of respiration in yellow-leaved seedlings depleted their stored nutrients

C

ATP serves as a common energy source for organisms because A it is the smallest energy molecule B it stores the least energy of any energy source C its energy can be easily transferred to do cellular work D it is extremely stable and can be stored in the cell for long periods of time E traces of it have been found in fossils of ancient organisms dating back to the beginning of life on Earth

C

Achondroplastic dwarfism is a dominant genetic trait that causes severe malformation of the skeleton. Homozygotes for this condition are spontaneously aborted (hence, the homozygous condition is lethal) but heterozygotes will develop to be dwarfed. Matthew has a family history of the condition, although he does not express the trait. Jane is an achondroplastic dwarf. Matthew and Jane are planning a family of several children and want to know the chances of producing a child with achondroplastic dwarfism. The probability that Matthew and Jane's first child will be an achondroplastic dwarf is A 0% B 25% C 50% D 75% E 100%

C

An African violet grower observes that genetically identical African violet plants growing near the walls of the greenhouse have white flowers, that plants growing farther away from the walls have pale blue flowers, and that plants growing nearest the center of the greenhouse have dark blue flowers. Which of the following best explains the differences in flower color of the African violets in the greenhouse? A Warmer temperatures result in genotypic alterations, which result in flower color differences. B The plants along the walls of the greenhouse are homozygous recessive and therefore have white flowers. C An enzyme responsible for flower color does not fold correctly in cooler temperatures, and the greenhouse is warmest in the center. D More light is available along the walls of the greenhouse, so the flowers need less pigment to absorb sunlight for photosynthesis.

C

An experiment to measure the rate of respiration in crickets and mice at 10oC and 25oC was performed using a respirometer, an apparatus that measures changes in gas volume. Respiration was measured in mL of O2 consumed per gram of organism over several five-minute trials and the following data were obtained. CR topic questions #3 According to the data, the crickets at 25oC have greater oxygen consumption per gram of tissue than do the crickets at 10oC. This trend in oxygen consumption is the opposite of that in the mice. The difference in trends in oxygen consumption among crickets and mice is due to their A relative size B mode of nutrition C mode of internal temperature regulation D mode of ATP production

C

An intermediate electron acceptor for oxidations that occur in both glycolysis and in Krebs cycle reactions A Cytochrome B FADH2 C NAD+ D NADP+ E Oxygen (O2)

C

Both myoglobin and hemoglobin are proteins that bind reversibly with molecular oxygen. The graph below shows the oxygen-binding saturation of each protein at different concentrations of oxygen. Chem of life #25 Which of the following statements is correct? A At 10 mm Hg partial pressure, hemoglobin binds oxygen but myoglobin does not. B At 20 mm Hg partial pressure, myoglobin and hemoglobin bind oxygen in equal amounts. C At 40 mm Hg partial pressure, myoglobin has a greater affinity for oxygen than hemoglobin has. D At 80 mm Hg partial pressure, myoglobin binds twice as much oxygen as hemoglobin binds.

C

Cancer can result from a variety of different mutational events. Which of the following is LEAST likely to result in the initiation of a cancerous tumor? A A receptor mutation results in activation of a cell-division pathway in the absence of the appropriate ligand. B A mutation results in the loss of the ability to produce a tumor-suppressor protein. C A defect in a cell-cycle checkpoint prevents a cell from entering the S phase. D At the anaphase checkpoint, separation of chromatids occurs without all centromeres being attached to kinetochore microtubules from both poles.

C

Cell communication is critical for the function of both unicellular and multicellular eukaryotes. Which of the following is likely true of cell signaling? A Cell signaling uses the highest molecular weight molecules found in living cells. B Cell signaling has largely been replaced by other cell functions in higher mammals. C Similar cell signaling pathways in diverse eukaryotes are evidence of conserved evolutionary processes. D Cell signaling functions mainly during early developmental stages.

C

Cell communication unit test #24 The graph above shows changes in glucagon and insulin secretions at different concentrations of blood glucose. Which of the following feedback mechanisms is best supported by the data? A A falling glucagon level causes a rise in the insulin level, which maintains equal amounts of both hormones in the blood. B A high glucagon level causes a rise in the insulin level, which maintains high levels of both hormones in the blood. C A low glucose level causes the release of glucagon, which stimulates the release of more glucose from tissues, which in turn lowers the amount of glucagon being released. D A low glucose level causes the release of insulin, which stimulates the release of more glucose from tissues, which in turn increases the amount of insulin being released.

C

Cell communication unit test #8 Insulin, a hormone secreted by pancreatic cells, stimulates glucose uptake in skeletal muscle cells by mobilizing glucose transporter proteins (GLUT4) to the plasma membrane. As depicted in Figure 1, binding of insulin to the insulin receptor triggers an intracellular signaling cascade in which certain molecules activate other molecules in a relay of the hormone signal to cell targets. One outcome of the signaling cascade is mobilization of GLUT4 from vesicle storage sites in the cytoplasm to sites at the cell surface, where GLUT4 allows glucose to enter the cell. In type 2 diabetes, the cellular response to insulin is disrupted, and individuals with type 2 diabetes cannot properly regulate their blood glucose levels. In an investigation of the insulin signaling pathway, samples of skeletal muscle were isolated from individuals who have type 2 diabetes and from individuals who do not. The results of several experiments that were performed on the muscle samples are shown in Figure 2, Figure 3, and Figure 4. Based on the experimental results, which of the following describes the most likely defect in muscle cells of patients with type 2 diabetes? A Insulin receptor proteins do not reach the cell surface. B Insulin does not activate its receptor. C IRS-1 activation is reduced at high insulin concentrations. D GLUT4 blocks glucose from entering cells.

C

Cell cycle practice questions #2 Researchers performed an experiment to determine the effect of certain genetic mutations on mitosis in tropical fruit fly embryos. They determined the percentage of cells in each of four phases of mitosis as shown in Figure 1. Figure 1. Percent of cells in phases of mitosis Which of the following patterns is shown by the data? A Mutant 1 cells are more similar to mutant 3 cells than to wild-type cells. B In wild-type cells, the percent of cells in anaphase is twice the amount of those in telophase C In mutant 3 cells, more time is spent in prophase/prometaphase than in the later stages of mitosis. D The percent of mutant 2 cells in anaphase is higher than that of mutant 1 cells.

C

Cell cycle questions #4 Figure 1 shows the number of chromosomes observed in an actively dividing human cell at each stage of cell division. Figure 1. Number of chromosomes in a human cell at different stages of cell division Which of the following presents a correct interpretation of the changes in chromosome number depicted in Figure 1 ? A DNADNA replication occurs between metaphase and anaphase, doubling the number of chromosomes. Between telophase and cytokinesis, the cell divides in two, with each cell receiving half of the replicated chromosomes. B New chromosomes formed during prophase are doubled during anaphase and are recombined before cytokinesis. C Chromosomes enter metaphase containing two chromatids attached by a centromere. During anaphase, the chromatids are separated, each becoming a chromosome. Cytokinesis distributes the chromosomes into two separate cells. D At anaphase a cell contains two identical copies of each chromosome, but following telophase, one of the copies is broken down into nucleotides.

C

Cells that contain only circular chromosomes are most probably which of the following? A Protist cells B Fungal cells C Bacterial cells D Plant cells E Animal cells

C

Directions: Each group of questions below concerns an experimental or laboratory situation or data. In each case, first study the description of the situation or data. Then choose the one best answer to each question following it. The following questions refer to the following information and graph. The data presented in the figure below are measurements of the rate of oxygen consumption at differing body masses in a species of fish. Each point represents measurements from a different fish. Measurements were taken at different temperatures. (○= 10°C,●= 15°C, □= 20°C, ■= 25°C.) CR quiz #25 The fact that each line on the graph rises from left to right means that A higher temperatures produce higher rates of metabolism B there were more large fish in the samples taken at high temperatures C larger fish consume more oxygen than smaller fish at all four temperatures D when measurements are taken for larger fish late in the day, observed values are higher E larger fish prefer to live at higher temperatures than do smaller fish

C

Directions: This group of questions consists of five lettered headings followed by a list of phrases or sentences. For each phrase or sentence, select the one heading to which it is most closely related. Each heading may be used once, more than once, or not at all. This group of questions refers to the following groups of biological compounds. (A) Proteins (B) Carbohydrates (C) Nucleic acids (D) Lipids (E) Steroids Used to carry the genetic code A Proteins B Carbohydrates C Nucleic acids D Lipids E Steroids

C

Feedback Practice questions #1 The epinephrine signaling pathway plays a role in regulating glucose homeostasis in muscle cells. The signaling pathway is activated by the binding of epinephrine to the beta-2 adrenergic receptor. A simplified model of the epinephrine signaling pathway is represented in Figure 1. Figure 1. A simplified model of the epinephrine signaling pathway in muscle cells Cyclic AMP phosphodiesterase is an enzyme that catalyzes the conversion of cyclic AMP to a different molecule. Which of the following best predicts the effect of inhibiting cyclic AMP phosphodiesterase in a muscle cell stimulated by epinephrine? A The concentration of cyclic AMPAMP will decrease because adenylyl cyclase will no longer be activated. B The G protein will diffuse out of the cell because it will no longer bind to the plasma membrane. C Phosphorylase kinase will remain active because protein kinase A will no longer be deactivated. D Glycolysis will stop because epinephrine signaling will no longer stimulate glycogen breakdown.

C

Figure 1. Formation of a peptide bond Which of the following best describes the formation of the bond shown in Figure 1 ? Test redo question 12 A An ionic bond is formed between a carbon atom of one amino acid and the nitrogen atom of the other amino acid. B An ionic bond is formed when the negative charge of an OHOH group is balanced by the positive charge of a hydrogen ion. C A covalent bond is formed between a carbon atom and a nitrogen atom along with the formation ofH2OH2O . D A covalent bond is formed that replaces the hydrogen bond between the OHOH group and the HH atom.

C

Figure 1. Formation of a peptide bond Which of the following best describes the formation of the bond shown in Figure 1 ? chem of life #17 A An ionic bond is formed between a carbon atom of one amino acid and the nitrogen atom of the other amino acid. B An ionic bond is formed when the negative charge of an OHOH group is balanced by the positive charge of a hydrogen ion. C A covalent bond is formed between a carbon atom and a nitrogen atom along with the formation ofH2OH2O . D A covalent bond is formed that replaces the hydrogen bond between the OHOH group and the HH atom.

C

Gregor Mendel's pioneering genetic experiments with pea plants occurred before the discovery of the structure and function of chromosomes. Which of the following observations about inheritance in pea plants could be explained only after the discovery that genes may be linked on a chromosome? A Pea color and pea shape display independent inheritance patterns. B Offspring of a given cross show all possible combinations of traits in equal proportions. C Most offspring of a given cross have a combination of traits that is identical to that of either one parent or the other. D Recessive phenotypes can skip a generation, showing up only in the parental and F2 generations.

C

Hereditary unit test # 25 The diagram above depicts a karyotype of an individual human. Which of the following statements concerning the karyotype in the diagram is true? A The diagram illustrates a genetic condition found in females. B The diagram indicates a mechanism for increasing genetic diversity in subsequent generations. C The diagram illustrates the results of nondisjunction during gamete formation. D The diagram indicates Down syndrome, a genetic condition.

C

Hereditary unit test #19 For following group of questions first study the description of the situation and diagram and then choose the one best answer to each question following it and fill in the corresponding oval on the answer sheet. The pedigree of a family with a history of a particular genetic disease is shown below. Squares represent males and circles represent females. Shaded symbols represent those who have the disease. If Individual 6 marries a woman with the disease, what is the probability that their first child will have the disease? A 0 B 25% C 50% D 75% E 100%

C

Hereditary unit test #24 The relative location of four genes on a chromosome can be mapped from the following data on crossover frequencies Which of the following represents the relative positions of these four genes on the chromosome? A ABCD B ADCB C CABD D CBAD E DBCA

C

Hereditary unit test #28 Directions: Each group of questions below concerns an experimental or a laboratory situation. In each case, first study the description of the situation. Then choose the one best answer to each question following it. In dogs, one pair of alleles determines coat color (dark and albino). Another pair of alleles determines hair length (short and long). Thus, each gamete will contain one of the coat-color alleles, C or c and one of the hair-length alleles, B or b. In repeated crosses of a specific dark, short-haired dog with an albino, long-haired dog, all the offspring were dark with short hair, as shown in cross I. However, in subsequent crosses of another dark, short-haired dog with a dark, long-haired dog, the ratios shown in cross II below were obtained. Which of the following is probably the genotype of the dark, short-haired parent in cross I? A CcBb B ccbb C CCBB D CCbb E ccBB

C

If ATP breakdown (hydrolysis) is inhibited, which of the following types of movement across cell membranes is also inhibited? A Movement of oxygen into a cell B Movement of water through aquaporins C Passage of a solute against its concentration gradient D Facilitated diffusion of a permeable substance

C

If chemical signals in the cytoplasm control the progression of a cell to the M phase of the cell cycle, then fusion of a cell in G1 with a cell in early M phase would most likely result in the A replication of chromosomes only in the G1 cell B exiting of both cells from the cell cycle and into the G0 phase C condensation of chromatin in preparation of nuclear division in both cells D transfer of organelles from the G1 cell to the cell in the M phase

C

In a certain signal transduction pathway, the binding of an extracellular molecule to a cell-surface protein results in a rapid increase in the concentration of cyclic AMP inside the cell. The cyclic AMP binds to and activates cytosolic enzymes that then activate other enzymes in the cell. Which of the following statements best describes the role of cyclic AMP in the signal transduction pathway? A It acts as a signaling molecule that passes the signal from the cell to other cells. B It acts as a receptor that carries the signal from outside the cell to inside the cell. C It acts as a second messenger that helps relay and amplify the signal within the cell. D It acts as a channel protein that transmits the signal across the cell's nuclear membrane.

C

In an experiment, a scientist isolates mitochondria from living cells and suspends them in two different buffered solutions. One solution is maintained at pH 4, while the other solution is maintained at pH 9. The scientist finds that mitochondria in the solution at pH 4 continue to produce ATP but those in the pH 9 solution do not. The results of the experiment can be used as evidence in support of which of the following scientific claims about mitochondrial activity? A Mitochondria in a cell-free environment are unable to convert thermal energy into ATP. B The electron transport chain pumps electrons from the cytosol to the mitochondrial matrix. C ATP production in mitochondria requires a hydrogen ion gradient that favors movement of protons into the mitochondrial matrix. D ATP synthase molecules change their orientation in relation to the proton gradient across the mitochondrial membrane.

C

Intravenous (IV) solutions administered to patients are normally isotonic. Which of the following is most likely if an IV of distilled water is administered to a patient? A The cells that are exposed to hypotonic solutions will shrink as a result of salt moving into the blood. B The liver will secrete additional bile salts into the blood to raise the tonicity of the administered fluid. C The cells that are exposed to hypotonic solutions will expand as water moves osmotically into the cells from the blood. D The patient's respiration rate will slow to compensate for the higher levels of circulating blood

C

Meiosis topic questions #2 #1 Table 1 shows the stage and number of cells and chromosomes per cell at the end of the stage in a 2n=24 organism. Table 1. Cell and chromosome count during selected phases of meiosis StageNumber of CellsNumber of Chromosomes per CellProphase I124Metaphase I124Anaphase I124Telophase I124Beginning of Prophase II212 Which of the following statements correctly describes the chromosomes in each daughter cell at the end of meiosis I? A Each daughter cell contains 12 chromatids. Each chromatid is one of two from a single chromosome with the other one of the pair found in the other daughter cell. B Each daughter cell contains 12 chromosomes, each composed of two chromatids. Since the chromosomes were randomly divided, one daughter cell may contain both of a pair of homologous chromosomes, while the other cell contains both of another pair of homologous chromosomes. C Each daughter cell contains 12 chromosomes, each composed of two chromatids. Each chromosome is one of a pair of homologous chromosomes from the parent cell, with the other homologue found in the other daughter cell. D Each daughter cell contains 24 separate chromatids. Since every two chromatids were originally joined, forming one homologous chromosome, the number of chromatids is divided by two to determine the number of chromosomes.

C

Meiosis topic questions #3 Which of the following questions about genetic diversity could most appropriately be answered by analysis of the model in Figure 1 ? A Does crossing-over generate more genetic diversity than the fusion of gametes does? B Does DNADNA methylation prevent independent assortment during metaphase IIII? C How does the independent assortment of the two sets of homologous chromosomes increase genetic diversity? D Do daughter cells that are not genetically identical to parent cells produce viable zygotes?

C

Notch is a receptor protein displayed on the surface of certain cells in developing fruit fly embryos. Notch's ligand is a membrane-bound protein called Delta that is displayed on the surface of adjacent cells. When Notch is activated by its ligand, the intracellular tail of the Notch protein becomes separated from the rest of the protein. This allows the intracellular tail to move to the cell's nucleus and alter the expression of specific genes. Which of the following statements best explains Delta's role in regulating cell communication through the Notch signaling pathway? A Delta transmits a chemical signal to all the cells of a developing embryo. B Delta allows the cells of a developing embryo to communicate without making direct contact. C Delta restricts cell communication to short distances within a developing embryo. D Delta determines which cells in a developing embryo express the gene that encodes the Notch protein.

C

Researchers studying cell cycle regulation in budding yeast have observed that a mutation in the CDC15 gene causes cell cycle arrest in telophase when the yeast cells are incubated at an elevated temperature. Which of the following statements best predicts the effect of the cell cycle arrest on proliferating yeast cells? A The yeast cells will transition out of G0G0 but will fail to complete the G1G1 phase. B The yeast cells will initiate mitosis but will fail to complete the G2G2 phase. C The yeast cells will replicate their chromosomes but will fail to complete cytokinesis. D The yeast cells will replicate their organelles but will fail to complete the S phase.

C

Students conducted a controlled experiment to investigate whether sawdust provides enough nutrients to support plant growth. The students separated ten nearly identical sunflower seedlings into two groups. They grew the seedlings in the first group in potting soil and the seedlings in the second group in sawdust composed mostly of cellulose. After twenty days, the students recorded observations about the seedlings in each group. The students' observations are presented in the table. Treatment GroupMean Seedling HeightObservationsSeedlings in potting soil18.5 cmThe leaves have a dark green color and are normal in size.Seedlings in sawdust4.8 cmThe leaves have a grayish color and are small in size. The observed differences between the groups most likely resulted from differences in the ability of the seedlings to produce which of the following monomers?chem of life # 15 A B C D

C

The Hedgehog protein (Hh) plays a critical role during a certain period of embryo development, but it normally has no role in adults except for the maintenance of adult stem cells. However, the Hedgehog protein has been detected in 70 percent of pancreatic cancer cell samples. As illustrated in the figures below, the Hedgehog protein binds to an integral embrane protein receptor known as Patched (Ptc), thus initiating a pathway of gene expression. When Hedgehog is absent, Ptc inhibits another protein known as Smoothened (Smo), which, in turn, blocks the activation of a group of proteins collectively known as the Hedgehog signaling complex (HSC). The inactivation is the result of proteolytic cleavage of one component of the HSC complex, a transcription factor known as Cubitus interruptus (Ci). When Hedgehog is present, it binds to Ptc, which prevents the inhibition of Smo by Ptc. The result is that Ci remains intact and can enter the nucleus, where it binds to and activates certain genes. Cell signaling quiz #9 One approach to treating patients with pancreatic cancer and other cancers in which the Hedgehog protein is detected is to modify the Hedgehog signaling pathway. Which of the following is the most useful approach? A Treating patients with a molecule that is structurally similar to Hedgehog and that will bind to and interact with Ptc in the same fashion as Hedgehog B Injecting patients with embryonic cells so that Hedgehog will bind to those cells instead of the cancer cells C Treating patients with a membrane-soluble compound that can bind to Smo and block its activity D Injecting patients with a preparation of purified membrane-soluble Ci that will enter the nuclei of the cancer cells and induce gene transcription

C

The carbohydrates glucose, galactose, and fructose have the same chemical formula (C6H12O6) but different structural formulas, as represented in the figure. Which of the following statements about glucose, galactose, and fructose is most likely true? A The carbohydrates have the same properties because they have the same number of carbon, hydrogen, and oxygen atoms. B The carbohydrates have the same properties because they each have a single carbon-oxygen double bond. C The carbohydrates have different properties because they have different arrangements of carbon, hydrogen, and oxygen atoms. D The carbohydrates have different properties because they have different numbers of carbon-carbon bonds.

C

The epinephrine signaling pathway plays a role in regulating glucose homeostasis in muscle cells. The signaling pathway is activated by the binding of epinephrine to the beta-2 adrenergic receptor. A simplified model of the epinephrine signaling pathway is represented in Figure 1. Cell signaling quiz review #5 Figure 1. A simplified model of the epinephrine signaling pathway in muscle cells Which of the following statements best describes the role of adenylyl cyclase in the epinephrine signaling pathway? A It converts a polymer to its monomer subunits. B It moves substances across the plasma membrane. C It accelerates the production of a second messenger. D It transfers phosphate groups from ATPATP to protein substrates.

C

The molecular structures of linoleic acid and palmitic acid, two naturally occurring substances, are shown in the figure. Based on the molecular structures shown in the figure, which molecule is likely to be solid at room temperature? Chem of life 10 A Linoleic acid, because the absence of carbon-carbon double bonds allows the molecules to pack closely together. B Linoleic acid, because the presence of carbon-carbon double bonds prevents the molecules from packing closely together. C Palmitic acid, because the absence of carbon-carbon double bonds allows the molecules to pack closely together. D Palmitic acid, because the presence of carbon-carbon double bonds prevents the molecules from packing closely together.

C

The sequences for two short fragments of DNA are shown above. Which of the following is one way in which these two segments would differ? A Segment 1 would not code for mRNA because both strands have T, a base not found in RNA. B Segment 1 would be more soluble in water than segment 2 because it has more phosphate groups. C Segment 1 would become denatured at a lower temperature than would segment 2 because A-T base pairs have two hydrogen bonds whereas G-C base pairs have three. D Segment 1 must be from a prokaryote because it has predominantly A-T base pairs.

C

Transcription review question #12 Figure 1. Model of process involved in gene expression. Which of the following best explains what strand XX represents? A A complementary RNARNA sequence, because it contains thymine B The coding strand in this process, because it is being read 3′3′ to 5′5′ C The antisense strand, because it is serving as a template D The pre‑mRNAmRNA, because it does not yet have a GTPGTP cap

C

Two different models of a living cell are represented in the figure. Of the two cells represented in the figure, which would likely be more efficient at exchanging substances with the surrounding environment? Cell size quiz prep #4 A Cell A, because it has the larger surface-area-to-volume ratio. B Cell A, because it has the smaller surface-area-to-volume ratio. C Cell B, because it has the larger surface-area-to-volume ratio. D Cell B, because it has the smaller surface-area-to-volume ratio.

C

Unit 5 practice questions # 20 A student carries out a genetics experiment with fruit flies to investigate the inheritance pattern of the white eye trait. The student crosses a homozygous white-eyed female with a wild-type male and records observations about the flies in the F1 generation. The student plans to use the F1 data to perform a chi-square goodness-of-fit test for a model based on an X-linked recessive pattern of inheritance. The student will use one degree of freedom and a significance level of p=0.05. The setup for the student's chi-square goodness-of-fit test is presented in Table 1. Table 1. Setup for the student's chi-square goodness-of-fit test PhenotypeObservedExpectedRed-eyed female5350White-eyed male4750 The student calculates a chi-square value of 0.36. Which of the following statements best completes the student's chi-square goodness-of-fit test? A The critical value is 0.05, and the student cannot reject the null hypothesis. B The critical value is 0.05, and the student can reject the null hypothesis. C The critical value is 3.84, and the student cannot reject the null hypothesis. D The critical value is 3.84, and the student can reject the null hypothesis.

C

Unit 5 practice questions #21 Trisomy 21 is a condition in which a child is born with an extra chromosome in pair 21. Researchers assessed the frequency of children born with trisomy 21 by age of the mothers at birth (maternal age) and primary cause of the error leading to trisomy 21. The findings are presented in Figure 1. Figure 1. Incidence and primary cause of trisomy 21 by maternal age-group Based on the data in Figure 1, which of the following is most likely the primary cause of the pattern of frequency of trisomy 21 births in the selected maternal age-groups? A At older maternal ages, there is an increase in the number of errors during mitosis, which leads to an increase in nondisjunction during egg production. B The incidence of nondisjunction errors in meiosis during sperm production is positively correlated with increasing maternal age. C At older maternal ages, the incidence of errors in meiosis during egg production increases, which leads to an increase in nondisjunction. D Errors in meiosis leading to nondisjunction are more likely to occur during meiosis II than during meiosis IIII.

C

Unit 5 practice questions #4 Red-green color blindness in humans is caused by a recessive allele located on the X chromosome. Figure 1 shows the potential offspring of a female who is red-green color-blind and a male with full-color vision. All of the possible male offspring would be color-blind, and all of the possible female offspring would have full-color vision. If during the production of male gametes an error in meiosis occurred, sperm containing both an X and a Y chromosome could be produced. Figure 1. Possible offspring of a female who is red-green color-blind and a male who has full-color vision. How would the extra chromosome affect the male offspring produced by the gamete? A None of the potential offspring would be male, because the potentially male zygote would have twoXX chromosomes, and the YY chromosome would be ignored. B The male offspring would all be red-green color-blind, because of interference from alleles on the YYchromosome. C The male offspring would have full-color vision, because of the presence of the extra XXchromosome. D There would be no change to the phenotypes of the possible offspring, because the extra XXchromosome would not be active.

C

Unit 5 practice questions #6 A model showing two possible arrangements of chromosomes during meiosis is shown in Figure 1. Figure 1. Two possible arrangements of chromosomes during meiosis Which of the following questions about genetic diversity could most appropriately be answered by analysis of the model in Figure 1 ? A Does crossing-over generate more genetic diversity than the fusion of gametes does? B Does DNADNA methylation prevent independent assortment during metaphase IIII? C How does the independent assortment of the two sets of homologous chromosomes increase genetic diversity? D Do daughter cells that are not genetically identical to parent cells produce viable zygotes?

C

Vertebrate immune responses involve communication over short and long distances. Which of the following statements best helps explain how cell surface proteins, such as MHC proteins and T cell receptors, mediate cell communication over short distances? A The proteins receive electrical signals from nerve cells. B The proteins leave the cell and travel in the bloodstream to other cells. C The proteins interact directly with proteins on the surfaces of other cells. D The proteins bind to molecules secreted by cells located in other parts of the body.

C

Which of the following best represents two different signaling pathways that share a second messenger? Cell signaling quiz #12 A B C D

C

Which of the following correctly explains where DNA replication will begin on the strand oriented 5'→3', reading from left to right? A DNADNA replication will be randomly initiated along the unwound portion of the DNADNA strand since base pairing will occur. B DNADNA replication cannot occur since there is already RNARNA base pairing with the template strand. C DNADNA replication will be initiated immediately to the left of the RNARNA, since DNADNA polymerase requires an RNARNA primer. D DNADNA replication will be initiated at the site of the topoisomerase since that is where DNADNA begins to uncoil.

C

Which of the following describes a metabolic consequence of a shortage of oxygen in muscle cells? A An increase in blood pH due to the accumulation of lactic acid B No ATP production due to the absence of substrate-level phosphorylation C A buildup of lactic acid in the muscle tissue due to fermentation D A decrease in the oxidation of fatty acids due to a shortage of ATP

C

Which of the following is responsible for the cohesive property of water? A Hydrogen bonds between the oxygen atoms of two adjacent water molecules B Covalent bonds between the hydrogen atoms of two adjacent water molecules C Hydrogen bonds between the oxygen atom of one water molecule and a hydrogen atom of another water molecule D Covalent bonds between the oxygen atom of one water molecule and a hydrogen atom of another water molecule E Hydrogen bonds between water molecules and other types of molecules

C

Which of the following scientific questions is most relevant to the model represented in the figure above? cells quiz #3 #8 A Is ATP required for the transportation of sugars across the outer mitochondrial membrane? B Do the types of phospholipids in a membrane affect the rate at which molecules enter a cell by passive diffusion? C Which molecular substance is actively transported across the plasma membrane? D How does temperature affect the movement of molecules into lysosomes?

C

(A) Glysolysis (B) Krebs cycle (citric acid cycle) (C) Calvin cycle (light-independent reactions of photosynthesis) (D) Light-dependent reactions of photosynthesis (E) Chemiosmosis Process in which O2 is released as a by-product of oxidation-reduction reactions A B C D E

D

A researcher claims that increased atmospheric carbon dioxide levels cause increased growth rates in plants. Which of the following statements best supports the researcher's claim? A Atmospheric carbon dioxide is produced by the burning of fossil fuels, which are formed from the remains of living organisms such as plants. B Atmospheric carbon dioxide is a byproduct of cellular respiration, which is a metabolic process that occurs in plants and other living organisms. C Atmospheric carbon dioxide typically enters plant leaves through stomata, which plants rely on for regulating gas exchange with the atmosphere. D Atmospheric carbon dioxide is the raw material for photosynthesis, which plants rely on for producing sugars and other organic compounds.

D

A tobacco plant can be made to express a gene from fireflies, resulting in the emission of light. Which of the following is the basis for this phenomenon? A Chloroplasts can be made to produce light if firefly proteins are injected into plant cells. B Fireflies and tobacco plants share a recent common ancestor. C Fireflies and tobacco plants are infected by the same kinds of bacteria. D Transcription and translation are fundamen-tally similar in both fireflies and tobacco plants. E Most enzymes in fireflies have the same amino acid sequence as the enzymes in tobacco plants.

D

According to the chemiosmotic theory (chemiosmotic coupling), the energy required to move protons from the mitochondrial matrix to the intermembrane space against a concentration gradient comes most directly from A photons of red or blue light B the hydrolysis of ATP C the breakdown of high-energy fatty acids in the mitochondrial matrix D electrons flowing along the electron transport chain E substrate-level phosphorylation

D

Achondroplastic dwarfism is a dominant genetic trait that causes severe malformation of the skeleton. Homozygotes for this condition are spontaneously aborted (hence, the homozygous condition is lethal) but heterozygotes will develop to be dwarfed. Matthew has a family history of the condition, although he does not express the trait. Jane is an achondroplastic dwarf. Matthew and Jane are planning a family of several children and want to know the chances of producing a child with achondroplastic dwarfism. The genotypes of Matthew and Jane are best represented as A Matthew: AA Jane:Aa B Matthew:Aa Jane:aa C Matthew:aa Jane:aa D Matthew:aa Jane:Aa E Matthew:Aa Jane:Aa

D

Antibiotics interfere with prokaryotic cell functions. Streptomycin is an antibiotic that affects the small ribosomal subunit in prokaryotes. Specifically, streptomycin interferes with the proper binding of tRNA to mRNA in prokaryotic ribosomes. Which of the following best predicts the most direct effect of exposing prokaryotic cells to streptomycin? A Amino acid synthesis will be inhibited. B No mRNA will be transcribed from DNA. C Posttranslational modifications will be prevented. D Synthesis of polypeptides will be inhibited.

D

Australian dragon lizards have a ZW sex-determination system. The male genotype is homogametic (ZZ), and the female genotype is heterogametic (ZW). However, all eggs incubated at temperatures above 32°C tend to develop into females. Which of the following best explains how the development of phenotypic female Australian dragon lizards with a ZZ genotype occurs when incubation temperatures are above 32°C? A Lizard embryos with a ZZZZ genotype cannot develop at temperatures above 32°C32°C. B At incubation temperatures above 32°C32°C, ZZ chromosomes are mutated into WW chromosomes. C At incubation temperatures above 32°C32°C, crossing over transfers genes from the WW chromosome to the ZZ chromosome, producing females. D Incubation temperatures above 32°C32°C inhibit the genes on the ZZ chromosome that produce proteins necessary for male development.

D

Both mitosis and meiosis begin with a parent cell that is diploid. Which of the following best describes how mitosis and meiosis result in daughter cells with different numbers of chromosomes? A In mitosis, the chromosomes consist of a single chromatid, which is passed to two haploid daughter cells. In meiosis, the chromosomes consist of two chromatids during the first round of division and one chromatid during the second round of division, resulting in two haploid daughter cells. B In mitosis, synapsis of homologous chromosomes results in four haploid daughter cells after one division. In meiosis, synapsis of homologous chromosomes occurs during the second division and results in four diploid daughter cells. C Mitosis produces one identical daughter cell after one round of division. Meiosis has two rounds of division and doubles the number of chromosomes in the second round of division, producing four diploid cells. D Mitosis produces two identical diploid daughter cells after one round of division. Meiosis produces four haploid daughter cells after two rounds of division.

D

Cancer cells behave differently than normal body cells. For example, they ignore signals that tell them to stop dividing. Which of the following conditions will most likely cause a normal body cell to become a cancer cell? A The environment already contains cancer cells. B The environment has an abundance of nutrients. C The environment lacks signals that would otherwise tell the cell to stop dividing. D The environment contains mutagens that induce mutations that affect cell-cycle regulator proteins.

D

Cell communication unit test #21 Based on the model of eukaryotic cell cycle regulation shown in the figure, which of the following best describes the effect of a drug that blocks the production of the mitotic cyclin? A The cell cycle would proceed uncontrollably, and the cell would become cancerous. B The G1 cyclin would functionally replace mitotic cyclin, and the cell would continue dividing normally. C DNA synthesis would be prevented, and the cell would stop dividing. D The cell would be prevented from entering mitosis, and the cell would stop dividing.

D

Cell communication unit test #25 Precise regulation of specific hormone levels is required for optimal sperm production in mammals, as summarized in the figure above. Anabolic-androgenic steroids (AAS) are synthetic variants of testosterone that are sometimes abused by persons who desire to enhance their athletic performance or alter their physique. Assuming that AAS function in the same way as naturally occurring testosterone, it is most likely that long-term abuse of AAS would A stimulate FSH secretion B stimulate testosterone production C stimulate LH secretion D reduce sperm production

D

Cell communication unit test #3 The diagram above illustrates feedback control as exerted by the hormone thyroxine. Following surgical removal of the thyroid gland, the level of TSH in the blood will increase. Which of the following best explains this increase? A Residual blood thyroxine, from prior to thyroid gland removal, will bind to cells in the anterior pituitary, signaling more TSH secretion. B Thyroxine will remain bound to thyroxine receptors on various body cells, and these body cells will secrete additional hormones that stimulate the anterior pituitary to secrete TSH. C Thyroxine that was stored in the anterior pituitary prior to thyroid gland removal will signal more TSH secretion. D A decrease in thyroxine levels means a loss of inhibition to the hypothalamus and anterior pituitary, leading to increased TSH secretion.

D

Cell cycle practice questions #1 The relative amount of DNA in a cell at various stages of the cell cycle is shown in Figure 1 . Figure 1. Amount of DNA per cell during different stages of the cell cycle, relative to the beginning of the G1 stage Which of the following best describes how the amount of DNA in the cell changes during M phase? A The amount of DNA doubles as the DNA is replicated. B The amount of DNADNA slightly increases as a result of new organelle synthesis. C The amount of DNADNA does not change while the cell grows. D The amount of DNADNA is halved as the cell divides into two daughter cells.

D

Cell cycle practice questions #7 Researchers determined the average amount of time that a particular type of eukaryotic cell spends in each phase of the cell cycle. The data collected by the researchers are represented in Figure 1. Figure 1. The average amount of time spent by a particular type of eukaryotic cell in each phase of the cell cycle Based on Figure 1, what percent of the time required to complete a full cycle do the cells typically spend in interphase? A 5%5% B 35%35% C 50%50% D 95%95%

D

Cell signaling quiz #17 The epinephrine signaling pathway plays a role in regulating glucose homeostasis in muscle cells. The signaling pathway is activated by the binding of epinephrine to the beta-2 adrenergic receptor. A simplified model of the epinephrine signaling pathway is represented in Figure 1. Figure 1. A simplified model of the epinephrine signaling pathway in muscle cells A researcher claims that the epinephrine signaling pathway controls a catabolic process in muscle cells. Which of the following statements best helps justify the researcher's claim? A Epinephrine is a signaling molecule that binds to a transmembrane protein. B The G protein in the epinephrine signaling pathway consists of three different subunits. C Phosphorylase kinase catalyzes the hydrolysis of ATPATP. D Glycogen phosphorylase catalyzes the conversion of glycogen to glucose-1-phosphate.

D

Cell signaling quiz #2 The figure above shows a model of a ligand precursor being cleaved to produce an active ligand that binds to a specific receptor. Which of the following is most likely to reduce the binding of the active ligand to its receptor? A A change in the cytoskeletal attachment of transmembrane proteins B The presence of a large amount of the precursor form of the ligand C An increase in the ratio of the number of unsaturated to the number of saturated fatty acid tails of the membrane lipids D A mutation in the receptor gene that causes a substitution of a charged amino acid for a nonpolar amino acid in the ligand binding site of the receptor

D

Cell signaling quiz #7 Steroid hormones, such as testosterone, pass through the plasma membrane and bind to an intracellular protein, as shown in the diagram below. The hormone-receptor complex then enters the nucleus, where it interacts with DNA to promote transcription of a specific gene. Based on the information presented, which of the following will also occur in response to steroid signaling? A Histone protein synthesis will increase because histones maintain the DNA in an optimal conformation for chromosome assembly. B Ribosome production will increase because ribosomes are specific for the mRNA with which they bind during translation. C DNA replication will increase as a result of the binding of the hormone-receptor complex to the DNA D Production of a specific mRNA will increase as a result of the binding of the hormone-receptor complex to the DNA

D

Cortisol is a hormone produced in response to stress, including starvation, in humans. Which of the following is most likely an immediate effect of a starvation-induced increase in cortisol secretion? A Increased activation of the immune system B Increased urine production by the kidneys C Increased bone and collagen formation D Increased mobilization of fatty acids from fat cells

D

Damaged tissue releases chemicals that activate platelets and stimulate the formation of blood clots. Which of the following predictions about the activity of platelets best describes a positive feedback mechanism? A Activated platelets release chemicals that inhibit blood clot formation. B Activated platelets release signaling molecules that inhibit cell division in damaged tissue. C Activated platelets constrict the blood vessels, stopping blood flow. D Activated platelets release chemicals that activate more platelets.

D

Ethylene causes fruits to ripen. In a signaling pathway, receptors activate transcription factors, which ultimately leads to ripening. Which of the following best supports the claim that ethylene initiates the signal transduction pathway that leads to ripening of fruit? A Ethylene is a simple gaseous molecule, which makes it easily detected by receptors. B Fruit will ripen in closed containers without exposure to air. C Ethylene synthesis is under both positive and negative feedback regulation. D Loss-of-function mutations in ethylene receptors result in changes to the ripening process.

D

Eye pigment in a particular strain of fly is determined by two genes. An autosomal gene that controls the color of the pigments in the eye has two alleles: a dominant allele ( R ) that results in red eyes and a recessive allele ( r ) that results in sepia eyes. A sex-linked gene that controls the expression of the colored pigments also has two alleles: a dominant allele ( T ) that allows for expression of the colored pigments and a recessive allele ( t ) that does not allow for expression of the colored pigments. Individuals without a T allele have white eyes regardless of the alleles of other eye-color genes. Which of the following represents a cross between a white-eyed female and a red-eyed male? A TtXRXR×ttXrY B TtXrXr×ttXRY C RRXTXT×RrXTY D RrXtXt×RrXTY

D

Figure 1 is a diagram of water molecules at the air-water interface at the surface of a pond. Figure 1. Alignment of water molecules at air-water interface Based on Figure 1, which of the following best describes how the properties of water at an air-water interface enable an insect to walk on the water's surface? A Covalent bonds between water molecules and the air above provide cohesion, which causes tiny bubbles to form under the feet of the insect. B Ionic bonds between molecules at the surface of the water provide an electric charge, which attracts the feet of the insect, keeping it on the surface. C Polar covalent bonds between molecules at the surface of the water provide adhesion, which supports the weight of the insect. D Hydrogen bonds between molecules at the surface of the water provide surface tension, which allows the water surface to deform but not break under the insect.

D

Figure 1 represents a common process that occurs in organisms. Figure 1. Structural formula for a common biological reaction Which of the following is an accurate description of the process shown in Figure 1 ? Chem of life #16 A The linking of amino acids with an ionic bond as an initial step in the protein synthesis process B The formation of a more complex carbohydrate with the covalent bonding of two simple sugars C The hydrolysis of amino acids with the breaking of covalent bonds with the release of water D The formation of a covalent peptide bond in a dehydration synthesis reaction

D

Figure 1 shows some relevant details of a model of how a deoxynucleotide, in this case dTMP, is added to a growing strand ofDNA. Figure 1. Model showing details of adding a deoxythymidine monophosphate (dTMP) nucleotide to a growing strand of DNA The features of this model provide evidence for which explanation of why all growing strands are synthesized in a 5′ to 3′ direction? A The two strands need to be antiparallel to bond properly. B Thymine and adenine would not bond properly if the strand grew from 3′3′ to 5′5′. C The translation of mRNAmRNA occurs in the 5′5′ to 3′3′ direction; therefore, the growing DNADNA strand must also grow in the 5′5′ to 3′3′ direction. D The phosphate group, attached to the 5′5′ carbon of the dTMPdTMP, forms a covalent bond with the oxygen atom attached to the 3′3′ carbon of the growing strand.

D

For following group of questions first study the description of the data and then choose the one best answer to each question following it and fill in the corresponding oval on the answer sheet. To study the actions of the enzyme catalase on hydrogen peroxide, students performed the following experiment. Catalase was extracted from potatoes by blending raw potatoes in a blender with cold distilled water. The filtrate was stored on ice. The following hydrogen peroxide solutions were made: 1 percent, 5 percent, 10 percent, and 15 percent. Filter paper disks were soaked in the catalase filtrate and dropped into beakers containing the various solutions. The activity of the enzyme was measured by the amount of time it took for the disks to float to the surface of the solution on the bubbles produced by the reaction. The following data were obtained. Energy Unit test # 10 Which of the following best describes why the disks rose to the surface faster in the more concentrated hydrogen peroxide solutions? A There was more enzyme present in the more concentrated solutions. B A greater amount of heat was generated in the more concentrated solutions. C The more concentrated solutions lowered the activation energy of the reaction. D The higher substrate concentrations in the more concentrated solutions speeded the reaction. E The density of the water was lower in the more concentrated solutions.

D

For following group of questions first study the description of the situation and diagrams and then choose the one best answer to each question following it and fill in the corresponding oval on the answer sheet. A student studied the effects of light intensity on oxygen production in green algae. The algae were suspended in water inside a sealed glass jar, and the jar was placed into a constant-temperature, lightproof box containing a light source. A probe was inserted into the jar to record the concentration of oxygen. The probe was connected to a recording device. The setup is shown below. The student decreased the intensity of the light hourly and recorded the corresponding changes in oxygen concentration. The graph below shows the results from the recording device. The rate of oxygen production equaled the rate of oxygen consumption during which of the following time periods? Photosynthesis quiz #6 A G to H B H to I C I to J D J to K E G to K

D

Hereditary Unit test #7 A student in a biology class crossed a male Drosophila melanogaster having a gray body and long wings with a female D. melanogaster having a black body and apterous wings. The following distribution of traits was observed in the offspring. Which of the following is supported by the data? A The alleles for gray body and long wings are dominant. B The alleles for gray body and long wings are recessive. C Genes for the two traits are located on two different chromosomes, and independent assortment occurred. D Genes for the two traits are located close together on the same chromosome, and crossing over occurred between the two gene loci.

D

Hereditary unit test #11 A scientist studying phenotypic variation in a species of butterfly observed that genetically identical caterpillars grown in similar cages but exposed to different colored lights developed into butterflies with differences in wing color and body size, as shown in Table 1. Table 1. Effect of Exposing Identical Caterpillars to Specific Colors of Light Phenotype of Adult ButterflyCaterpillars Exposed to Red LightCaterpillars Exposed to Blue LightWing colorDarkerLighterBody sizeSmallerLarger Which of the following best explains the cause of the phenotypic variation observed in the butterflies? A Different mutations occurred in the caterpillars that were exposed to different colors of light. B The energy used to grow a larger body results in butterflies with lighter colored wings. C Individual caterpillars evolved adaptations to survive in each of the light conditions they were exposed to. D There was differential gene expression of wing color and body size in response to the colors of light the caterpillars were exposed to.

D

Hereditary unit test #17 A male fruit fly (Drosophila melanogaster) with red eyes and long wings was mated with a female with purple eyes and vestigial wings. All of the offspring in the F1 generation had red eyes and long wings. These F1 flies were test crossed with purple-eyed, vestigial-winged flies. Their offspring, the F2 generation, appeared as indicated below. If in the F1 and F2 generations the same characteristics appeared in both males and females, it would be safe to assume that these traits for eye color and wing length A are sex-linked B vary in dominance according to sex C are sex-influenced characteristics D are autosomal characteristics E follow the Mendelian rule of independent assortment

D

Hereditary unit test #3 In the pedigree above, circles represent females, squares represent males, and shaded figures represent individuals expressing a specific trait. The expression of this trait is most likely due to which of the following? A Sex-linked dominant inheritance B Sex-linked recessive inheritance C Autosomal dominant inheritance D Autosomal recessive inheritance E A codominant relationship of a single pair of alleles

D

Hereditary unit test #5 In the pedigree above, squares represent males and circles represent females. Individuals who express a particular trait are represented by shaded figures. Which of the following patterns of inheritance best explains the transmission of the trait? A Sex-linked dominant B Sex-linked recessive C Autosomal recessive D Autosomal dominant E Incompletely dominant

D

Hereditary unit test #6 A series of crosses is performed with fruit flies (Drosophila melanogaster) to examine inheritance of the genes vestigial (vg) andcinnabar (cn). The recessive vg allele causes small, malformed wings called vestigial wings. The recessive cn allele causes bright-red eyes called cinnabar eyes. In the first cross, a female with wild-type wings and eyes is mated with a male with vestigial wings and cinnabar eyes. All the F1individuals have wild-type wings and eyes. In the second cross, female F1 flies are mated with males with vestigial wings and cinnabar eyes. The phenotypes of 500 F2 individuals are shown in the table Which of the following is the most likely explanation of the results? A The two genes are located on two different chromosomes. B The two genes are sex-linked. C The two genes are located on mitochondrial DNA. D The two genes are linked on an autosome.

D

In flowering plants, plasmodesmata are narrow channels through cell walls that connect the cytoplasms of adjacent cells. An explanation of how plant cells communicate across cell walls will most likely refer to the diffusion through plasmodesmata of which of the following? A Membrane-bound organelles B Condensed, duplicated chromosomes C Branched polysaccharides D Small, water-soluble molecules

D

Item 3 A spherical bacterial cell has a radius of 3μm. The human egg cell has a radius of 100μm. Which statement correctly indicates the cell that is able to more efficiently exchange materials with the external environment and provides a correct explanation? A The egg cell, because it has the smallest surface-to-volume ratio. B The egg cell, because it has the largest surface-to-volume ratio. C The bacterial cell, because it has the smallest surface-to-volume ratio. D The bacterial cell, because it has the largest surface-to-volume ratio.

D

Polypeptides are continuously being formed and degraded. One of these processes is shown. Figure 1. Polypeptide reaction Which statement is the most accurate description of the reaction shown in Figure 1? Test Redo question 10 A It represents monomers linked by dehydration synthesis. B It represents a polypeptide chain that folds to form the tertiary structure. C It represents a polypeptide chain that is denatured into the primary structure. D It represents a polypeptide chain that is broken down through a hydrolysis reaction.

D

Saccharomyces cerevisiae is a diploid yeast species that can reproduce either sexually or asexually. An experiment was performed to induce mitotically dividing S. cerevisiae cells in G2 to undergo meiosis. Which of the following best describes the steps these cells will follow to form gametes? A The first division will result in crossing over between homologous chromosomes, and the second division will reduce the original number of chromosomes by half in the daughter cells. B The first division will reduce the number of chromosomes by half for each daughter cell, and the second division will result in each daughter cell having one-fourth of the original number of chromosomes. C The first division will move single chromatids to each daughter cell, and the second division will double the number of chromosomes in each daughter cell. D The first division will reduce the number of chromosomes by half for each daughter cell, and the second division will move single chromatids to each daughter cell.

D

The CFTR protein is made up of 1,480 amino acids linked together in a chain. Some humans produce a version of the CFTRprotein in which phenylalanine (an amino acid) has been deleted from position 508 of the amino acid chain. Which of the following best predicts how the amino acid deletion will affect the structure of the CFTR protein? A It will have no observable effect on the structure of the CFTRCFTR protein. B It will affect the primary structure of the CFTRCFTR protein, but the other levels of protein structure will not be affected. C It will affect the secondary and tertiary structures of the CFTRCFTR protein, but the primary structure will not be affected. D It will affect the primary, secondary, and tertiary structures of the CFTRCFTR protein.

D

The epinephrine signaling pathway plays a role in regulating glucose homeostasis in muscle cells. The signaling pathway is activated by the binding of epinephrine to the beta-2 adrenergic receptor. A simplified model of the epinephrine signaling pathway is represented in Figure 1. Cell signaling quiz review # 7 Figure 1. A simplified model of the epinephrine signaling pathway in muscle cells A researcher claims that the epinephrine signaling pathway controls a catabolic process in muscle cells. Which of the following statements best helps justify the researcher's claim? A Epinephrine is a signaling molecule that binds to a transmembrane protein. B The G protein in the epinephrine signaling pathway consists of three different subunits. C Phosphorylase kinase catalyzes the hydrolysis of ATPATP. D Glycogen phosphorylase catalyzes the conversion of glycogen to glucose-1-phosphate.

D

The figure shows a model of the exchange of matter between the organisms that live together in an aquarium. The model includes matter exchange between plants, fish, and bacteria. The bacteria are represented as rod-shaped organisms living in the gravel at the bottom of the aquarium. Which of the following statements best describes how molecules released by the fish become nutrients for the plants? A The carbon dioxide molecules released by the fish are converted by the bacteria to oxygen atoms, which are used by the plants to make water molecules. B The oxygen molecules released by the fish are converted by the bacteria to ammonia molecules, which are used by the plants to make lipids and fatty acids. C The nitrites released by the fish are converted by the bacteria to carbon dioxide molecules, which are used by the plants to make carbohydrates. D The ammonia molecules released by the fish are converted by the bacteria to nitrates, which are used by the plants to make proteins and nucleic acids. Answer D Correct. As shown in the figure, the ammonia molecules released by the fish are converted by the bacteria to nitrites and nitrates. The plants absorb the nitrates, which are nitrogen-containing nutrients that the plants use to make the amino acids and nucleotides needed for building proteins and nucleic acids. Related Content & Skills Topic1.2 Skill

D

The following questions refer to the following diagram. For each phrase or sentence, select the labeled part to which it is most closely related. Each option may be used once, more than once, or not at all for each group. Cells quiz #3 #10 Site of transport of materials into and out of the cell A B C D E

D

The manner in which several different ions and molecules move through a cell membrane is shown in the diagram above. For each ion or molecule, the relative concentration on each side of the membrane is indicated. Which of the following accurately describes one of the movements taking place? cells quiz #3 #9 A Glucose is transported into the cell by active transport. B Na+ is transported into the cell by active transport. C The movement of glucose through the membrane requires ATP hydrolysis. D Na+ transport out of the cell requires ATP hydrolysis.

D

This group of questions refers to the probabilities below. Assume that the alleles referred to all assort independently. (A) 0 (B) 1/16 (C) 1/4 (D) 1/2 (E) 3/4 Probability that the genotype Aa will be produced by the parents Aa x Aa A 0 B 1/16 C 1/4 D 1/2 E 3/4

D

Transcription review questions # 13 Which of the following best describes an event during step 2 in the simplified model above? A new RNA molecule is synthesized using a DNA template. A A new polypeptide is synthesized using an RNA template. B Thymine nucleotides in an RNA molecule are replaced with uracil nucleotides. C Noncoding sequences are removed from a newly synthesized RNA molecule.

D

Transcription review questions #14 Figure 1. Cellular process involving nucleic acids Which of the following best explains the process represented by Figure 1 ? The synthesis of mRNA in the 5′ to 3′ direction from DNA A The modification of a protein to produce a functional form of that protein B The translation of an mRNA molecule into a polypeptide C The enzyme-regulated processing of pre‑mRNA into mature mRNA

D

Unit 5 Practice questions #12 In order to determine the effects of age on the accumulation of mitochondrial mutations, mitochondrial DNA samples from young mice (3 months) and old mice (30 months) were observed for mutations. Table 1. Number of mitochondrial mutations in 106 base pairs MouseYoungOldA1B1C3D2E4F10G15H12I9J11 Which of the following is a correct analysis of this data set? A There is an increase in the mean number of mutations for the two age groups of 9.0 mutations per106106 base pairs. This is more critical in male mammals since mitochondria are paternally inherited. B There is an increase in the mean number of mutations for the two age groups of 9.0 mutations per106106 base pairs. This is more critical in female mammals since mitochondria are maternally inherited. C There is an increase in the mean number of mutations for the two age groups of 9.2 mutations per106106 base pairs. This is more critical in male mammals since mitochondria are paternally inherited. D There is an increase in the mean number of mutations for the two age groups of 9.2 mutations per106106 base pairs. This is more critical in female mammals since mitochondria are maternally inherited.

D

Unit 5 Practice questions #14 Figure 1 illustrates the X and Y chromosomes during meiosis I and meiosis II of normal spermatogenesis in a mammal species. Figure 1. X and Y chromosomes during meiosis I and meiosis II If the normal spermatogenesis is disrupted, the gametes can have different chromosomes than expected. Which of the following is the most likely cause of one of the four gametes having two X chromosomes and one having neither an X nor a Y chromosome? A Nondisjunction of the chromosomes during meiosis II B Nondisjunction of both the XX and YY chromosomes during meiosis IIII C Nondisjunction of the YY chromosome during meiosis IIII D Nondisjunction of the XX chromosome during meiosis II

D

Unit 5 Practice questions #3 A gene that influences coat color in domestic cats is located on the X chromosome. A female cat that is heterozygous for the gene (XBXO) has a calico-colored coat. In a genetics experiment, researchers mate a calico-colored female cat (XBXO) with an orange-colored male cat (XOY) to produce an F1 generation. The researchers record observations for the cats in the F1 generation and plan to use the data to perform a chi-square goodness-of-fit test for a model of X-linked inheritance. The data for the chi-square goodness-of-fit test are presented in Table 1. The researchers calculate a chi-square value of 4.6 and choose a significance level of p=0.05. Which of the following statements best completes the chi-square goodness-of-fit test? A The null hypothesis can be rejected because the chi-square value is greater than the critical value. B The null hypothesis can be rejected because the chi-square value is less than the critical value. C The null hypothesis cannot be rejected because the chi-square value is greater than the critical value. D The null hypothesis cannot be rejected because the chi-square value is less than the critical value.

D

Unit 5 practice questions # 17 In fruit flies, purple eyes and ebony body are traits that display autosomal recessive patterns of inheritance. In a genetics experiment, students cross wild-type flies with flies that have purple eyes and ebony bodies. The students observe that all the flies in the F1 generation have normal eyes and a normal body color. The students then allow the F1 flies to mate and produce an F2generation. The students record observations about the flies in the F2 generation and use the data to perform a chi-square goodness-of-fit test for a model of independent assortment. The setup for the students' chi-square goodness-of-fit test is presented in Table 1. Table 1. The students' chi-square goodness-of-fit test for a model of independent assortment PhenotypeObservedExpectedNormal eyes, normal body187171Normal eyes, ebony body4957Purple eyes, normal body4157Purple eyes, ebony body2719 The students choose a significance level of p=0.01. Which of the following statements best completes the next step of the chi-square goodness-of-fit test? A The calculated chi-square value is 2.11, and the critical value is 7.82. B The calculated chi-square value is 2.11, and the critical value is 11.35. C The calculated chi-square value is 10.48, and the critical value is 7.82. D The calculated chi-square value is 10.48, and the critical value is 11.35.

D

Unit 5 practice questions #15 Pigeons demonstrate ZW sex determination, such that a ZZ genotype produces a male and a ZW genotype produces a female. The gene for feather color is located on the Z chromosome, and the red allele is dominant over the brown allele. Three crosses between brown male pigeons and red female pigeons were performed, and the results are shown below. Table 1. Offspring from three separate crosses of a brown male pigeon and a red female pigeon Number of OffspringPhenotypeCross 1Cross 2Cross 3Red1197Brown91113 Which of the following is the mean number of male offspring produced by the three crosses? A 27 B 20 C 11 D 9

D

Unit 5 practice questions #18 In fruit flies, sepia eyes and ebony body are traits that display autosomal recessive patterns of inheritance. To investigate whether the traits are genetically linked, students cross wild-type flies with a line of flies that have sepia eyes and ebony bodies. The students observe that all the flies in the F1 generation have normal eyes and normal bodies. The students allow the flies in the F1generation to mate and produce an F2 generation. The students then record observations for the flies in the F2 generation and use the data to perform a chi-square goodness-of-fit test for a model of independent assortment. The setup for the chi-square goodness-of-fit test is presented in Table 1. Table 1. Setup for the students' chi-square goodness-of-fit test PhenotypeObservedExpectedNormal eyes, normal body231279Normal eyes, ebony body8693Sepia eyes, normal body9793Sepia eyes, ebony body8231 The students calculate a chi-squared value of 92.86 and compare it with a critical value of 7.82. Which of the following best completes the chi-square goodness-of-fit test? A The null hypothesis cannot be rejected, and the students should conclude that the data fit a model of independent assortment. B The null hypothesis cannot be rejected, and the students should conclude that the data may have resulted from genetic linkage. C The null hypothesis can be rejected, and the students should conclude that the data fit a model of independent assortment. D The null hypothesis can be rejected, and the students should conclude that the data may have resulted from genetic linkage.

D

Unit 5 practice questions #2 A scientist studying phenotypic variation in a species of butterfly observed that genetically identical caterpillars grown in similar cages but exposed to different colored lights developed into butterflies with differences in wing color and body size, as shown in Table 1. Table 1. Effect of Exposing Identical Caterpillars to Specific Colors of Light Phenotype of Adult ButterflyCaterpillars Exposed to Red LightCaterpillars Exposed to Blue LightWing colorDarkerLighterBody sizeSmallerLarger Which of the following best explains the cause of the phenotypic variation observed in the butterflies? A Different mutations occurred in the caterpillars that were exposed to different colors of light. B The energy used to grow a larger body results in butterflies with lighter colored wings. C Individual caterpillars evolved adaptations to survive in each of the light conditions they were exposed to. D There was differential gene expression of wing color and body size in response to the colors of light the caterpillars were exposed to.

D

What is the expected percent change in the DNA content of a typical eukaryotic cell as it progresses through the cell cycle from the start of the G1 phase to the end of the G2 phase? A −100%−100% B −50%−50% C +50%+50% D +100%+100%

D

What most likely causes the trends in oxygen concentration shown in the graph above? Photosynthesis quiz #1 A The water becomes colder at night and thus holds more oxygen. B Respiration in most organisms increases at night. C More organisms are respiring at night than during the day. D Photosynthesis produces more oxygen than is consumed by respiration during the day.

D

When hydrogen ions are pumped out of the mitochondrial matrix, across the inner mitochondrial membrane, and into the space between the inner and outer membranes, the result is A damage to the mitochondrion B the reduction of NAD C the restoration of the Na-K balance across the membrane D the creation of a proton gradient E the lowering of pH in the mitochondrial matrix

D

Which of the following conclusions is most clearly supported by the representations of nucleic acid #1 and nucleic acid #2 ?Chem of life #14 A Nucleic acid #1#1 contains only purines, whereas nucleic acid #2#2 contains only pyrimidines. B Nucleic acid #1#1 contains the sugar ribose, whereas nucleic acid #2#2 contains the sugar deoxyribose. C Nucleic acid #1#1 contains positively charged phosphate groups, whereas nucleic acid #2#2 does not. D Nucleic acid #1#1 contains adenine-thymine base pairs, whereas nucleic acid #2#2 does not.

D

Which of the following is most directly responsible for water's unique properties? A It contains oxygen atoms. B It contains hydrogen atoms. C It is an ionic compound. D It forms hydrogen bonds. E It is nonpolar.

D

tRNA molecules deliver amino acids to ribosomes where the amino acid next A forms an ionic bond with another amino acid B is deaminated and catabolized C becomes an anticodon D is covalently bonded to a growing polypeptide E forms hydrogen bonds with esters

D

(A) Glysolysis (B) Krebs cycle (citric acid cycle) (C) Calvin cycle (light-independent reactions of photosynthesis) (D) Light-dependent reactions of photosynthesis (E) Chemiosmosis Process found in both photosynthesis and cellular respiration A B C D E

E

For following group of questions first study the description of the data and then choose the one best answer to each question following it and fill in the corresponding oval on the answer sheet. To study the actions of the enzyme catalase on hydrogen peroxide, students performed the following experiment. Catalase was extracted from potatoes by blending raw potatoes in a blender with cold distilled water. The filtrate was stored on ice. The following hydrogen peroxide solutions were made: 1 percent, 5 percent, 10 percent, and 15 percent. Filter paper disks were soaked in the catalase filtrate and dropped into beakers containing the various solutions. The activity of the enzyme was measured by the amount of time it took for the disks to float to the surface of the solution on the bubbles produced by the reaction. The following data were obtained. Energy Unit test #9 If the potato solution was boiled for 10 minutes and cooled for 10 minutes before being tested, the average time for the disks to float to the surface of the hydrogen peroxide solution would be A less than 1 second B 5 seconds C 10 seconds D 30 seconds E more than 30 seconds

E

If a segment of DNA is 5'-TAC GAT TAG-3', the RNA that results from the transcription of this segment will be A 3'-TAC GAT TAU-5' B 3'-ATG CTA ATA-5' C 3'-UAC GAU UAG-5' D 3'-AAC GAU UAA-5' E 3'-AUG CUA AUC-5'

E

Item 8 If a segment of DNA is 5'-TAC GAT TAG-3', the RNA that results from the transcription of this segment will be A 3'-TAC GAT TAU-5' B 3'-ATG CTA ATA-5' C 3'-UAC GAU UAG-5' D 3'-AAC GAU UAA-5' E 3'-AUG CUA AUC-5'

E

The following questions refer to an experiment that is set up to determine the relative volume of O2 consumed by germinating and nongerminating (dry) pea seeds at two different temperatures. The change in volume is detected by using a respirometer over a given period of time. The data are given below. Energy Unit test #28 Which of the following conclusions is best supported by the data? A Nongerminating pea seeds have a higher rate of respiration than germinating pea seeds do. B Light is required for pea seed germination. C In the nongerminating pea seeds, oxygen consumption is directly proportional to oxygen concentration. D Less carbon dioxide is produced by germinating pea seeds at 26ºC than at 10ºC. E In pea seeds an increase in temperature results in an increase in oxygen consumption.

E

The synthesis of protein or carbohydrate polymers always produces which of the following as a byproduct? A ATP B Oxygen C Carbon dioxide D Urea E Water

E

Transcription review questions #2 . . . . glycine-serine-glycine . . . Which of the following DNA strands will code for the amino acid sequence shown above? . . . ACTCCTTCT . . . A . . . TCTCCGTCG . . . B . . . CCGTCGACT . . . C . . . CCTCCGTCG . . . D . . . CCTTCGCCT . . .

E

Which of the following best describes the function of the coenzymes NAD+ and FAD in eukaryotic cellular respiration? A They participate in hydrolysis reactions by accepting protons from water molecules. B They participate directly in the phosphorylation of ADP to ATP. C They serve as final electron acceptors in the electron transport chain. D They aid vitamins such as niacin in the breakdown of glucose. E They accept electrons during oxidation-reduction reactions.

E

Which of the following is an important difference between light-dependent and light-independent reactions of photosynthesis? A The light-dependent reactions occur only during the day; the light-independent reactions occur only during the night. B The light-dependent reactions occur in the cytoplasm; the light-independent reactions occur in chloroplasts. C The light-dependent reactions utilize CO2 and H2O; the light-independent reactions produce CO2and H2O. D The light-dependent reactions depend on the presence of both photosystems I and II; the light-independent reactions require only photosystem I. E The light-dependent reactions produce ATP and NADPH; the light-independent reactions use energy stored in ATP and NADPH.

E

Which of the following statements about mitochondrial chemiosmosis is NOT true? A A proton gradient is established across the inner membrane of the mitochondrion. B The potential energy released from the mitochondrial proton gradient is used to produce ATP. C The mitochondrial proton gradient provides energy for muscle contraction. D Proteins embedded in the inner mitochondrial membrane play an important role in ATP synthesis. E Heat energy is required to establish the electron transport chain.

E


Related study sets

CH 2: Factors influencing Child Health

View Set

RN Concept-Based Assessment Level 2 Online Practice B

View Set

ACCTG 101 - Chapter 5 - Smart learning

View Set

Word Parts: Suffixes and Prefixes Quiz

View Set